MedSurg GI Ch 47,48,49, 50, ATI, PrepU

Pataasin ang iyong marka sa homework at exams ngayon gamit ang Quizwiz!

A client with a peptic ulcer is about to begin a therapeutic regimen that includes a bland diet, antacids, and famotidine (Pepcid). Before the client is discharged, the nurse should provide which instruction?

"Avoid aspirin and products that contain aspirin."

A nurse is teaching a client with gastritis about the need to avoid the intake of caffeinated beverages. The client asks why this is so important. Which of the following explanations from the nurse would be most accurate?

"Caffeine stimulates the central nervous system and thus gastric activity and secretions, which need to be minimized to promote recovery."

A client admitted for treatment of a gastric ulcer is being prepared for discharge on antacid therapy. Discharge teaching should include which instruction?

"Continue to take antacids even if your symptoms subside."

discharge teaching for client with mild Diverticulitis, what statements indicates understanding of teaching?

"I should Eat foods that are LOW fiber"

discharge teaching for pt with gastritis and prescription for Famotidine which statement indicated teaching was effective

"I should Make sure the water i drink is filtered"- food and water should have NO contaminants

discharge teaching for client with GERD, what statement indicates teaching was effective?

"I will Decrease the amount of carbonated beverages i drink"

A nurse is performing discharge teaching with a client who had a total gastrectomy. Which statement indicates the need for further teaching?

"I will have to take vitamin B12 shots up to 1 year after surgery."

A client with a peptic ulcer is diagnosed with Heliobacter pylori infection. The nurse is teaching the client about the medications prescribed, including metronidazole (Flagyl), omeprazole (Prilosec), and clarithromycin (Biaxin). Which statement by the client indicates the best understanding of the medication regimen?

"The medications will kill the bacteria and stop the acid production."

A patient with end-stage liver disease who is scheduled to undergo a liver transplant tells the nurse, "I am worried that my body will reject the liver." Which of the following statements is the nurse's best response to the patient? a) "You will need to take daily medication to prevent rejection of the transplanted liver. The new liver has a good chance of survival with the use of these drugs." b) "It is easier to get a good tissue match with liver transplants than with other types of transplants." c) "The problem of rejection is not as common in liver transplants as in other organ transplants." d) "You would not be scheduled for a transplant if there was a concern about rejection."

"You will need to take daily medication to prevent rejection of the transplanted liver. The new liver has a good chance of survival with the use of these drugs." Rejection is a primary concern. A transplanted liver is perceived by the immune system as a foreign antigen. This triggers an immune response, leading to the activation of T lymphocytes that attack and destroy the transplanted liver. Immunosuppressive agents are used as long-term therapy to prevent this response and rejection of the transplanted liver. These agents inhibit the activation of immunocompetent T lymphocytes to prevent the production of effector T cells. Although the 1- and 5-year survival rates have increased dramatically with the use of new immunosuppressive therapies, these advances are not without major side effects. The other statements are inaccurate or will not decrease the patient's anxiety.

client with Hepatic Encephalopathy asks nurse if she can have a larger portion of beef for dinner, what response by the nurse is appropriate

"you should Limit your animal protein intake. Can i get you a veggie burger instead?"

discharge teaching for pt with Hepatitis C, what statement indicates understanding of teaching?

'I will Avoid medications containing ACETAMINOPHEN" - Hepatotoxic

peptic ulcer disease diagnostic test

-(EGD) esophagogastroduodenoscopy -h pylori testing -hgb, hct

pancreatitis risk factors & causes

-Alcohol -bile tract disease -GI surgery

GERD diagnostic tests

-EGD -esophageal manometry -24 hr ambulatory esophageal pH monitoring

hepatitis diagnostic tests

-ELISA (hep c) -total bilirubin elevated -liver biopsy

cholecystitis & cholethiasis diagnostic tests

-HIDA hepatobiliary scan -PTC percutaneous transhepatic cholangiography -elevated wbc

cirrhosis complications

-PSE portal systemic encephalopathy -esophageal varices -acute graft rejectiion

pancreatitis diagnostic tests

-WBC elevated -Decreased calcium & magnesium -CT

irritable bowel syndrome s&s

-abdomen cramping/pain -diarrhea/constipation -belching (burp)

inflammatory bowel disease s&s

-abdominal pain/cramping -fever -abdominal distention

GERD complications

-aspiration -esophagus cancer -asthma exacerbations

inflammatory bowel disease risk factors/causes

-autoimmunity -genetics -diet low in fiber

irritable bowel syndrome treatment

-avoid caffeine -alsoetron (lotronex) -lubiprostone (amitiza)

cholecystitis & cholethiasis treatment

-bile acids: chenodiol -ESWL extracorporeal shock wave lithotripsy -cholecystectomy

cholecystitis & cholethiasis complications

-bile duct obstruction -bile peritonitis -PCS postcholecystectomy

inflammatory bowel disease complications

-bleeding -fluid & electrolyte imbalance -peritonitis

intestinal obstruction

-blockage that keeps food or liquid from passing through your small intestine or large intestine (colon) -mechanical 90% -nonmechanical 10%

hepatitis complications

-chronic hepatitis -fluminating hepatitis -cirrhosis of the liver

esophageal varices risk factors & causes

-cirrhosis -portal hypertension ( increased amount of pressure in the portal vein) -viral hepatitis

inflammatory bowel disease diagnostic test

-colonoscopy -occult blood stool -barium enema

intestinal obstruction diagnostic test

-ct scan -endoscopy -x ray

intestinal obstruction complications

-dehydration -electrolyte imbalance -metabolic alkalosis

esophageal varices diagnostic tests

-endoscopy -liver function test -TIPS Transjugular Intraheptic Portosystemic Shunt

cirrhosis s&s

-fatigue -wt loss -confusion/difficulty thinking

hepatitis s&s

-fever -vomiting -dark colored urine

cholecystitis & cholethiasis risk factors & causes

-gallstones (cholelithiasis) -high fat diet -type 1 DM

gastritis complications

-gastric bleeding -pernicious anemia -dumping syndrome (weakness, abdominal discomfort, and sometimes abnormally rapid bowel evacuation)

peptic ulcer disease cause & risk factors

-h pylori infection -NSAID & corticosteriod use -smoking -severe stress

gastritis risk factors & causes

-h.pylori -excessive stress -autoimmune disease

esophageal varices s&s

-hypotension -tachycardia

pancreatitis complications

-hypovolemia -pancreatic infection -coagulation defects

cholecystitis & cholethiasis

-inflammation of the gallbladder wall

cholecystitis & cholethiasis s&s

-jaundice -sharp pain (in upper quadrant) -steatorrhea (fatty stools)

cirrhosis diagnostic tests

-liver biopsy -EGD -elevated bilirubin

intestinal obstruction risk factors/causes

-mechanical: hernia, tumors -nonmechanical: neurogenic/vascular disorders

hepatitis treatment

-medications -high carb, high cal, low-mod fat, low-mod protein -limit client activity

cirrhosis risk factors & causes

-necrotic injury -alcohol abuse -chronic viral hepatitis

intestinal obstruction treatment

-ng tube -surgery: exploratory laparotomy -octreotide (sandostatin)

GERD risk factors & causes

-obesity -hiatal hernia (part of the stomach pushes up through the diaphragm muscle) -NG tube

intestinal obstruction s&S

-obstipation (severe or complete constipation) -abdominal distention -high pitch bowel sounds

cirrhosis treatment

-paracentesis -surgical bypass -liver transplant

peptic ulcer disease complications

-perforation/hemorrhage -pernicious anemia -dumping syndrome (weakness, abdominal discomfort, and sometimes abnormally rapid bowel evacuation)

GERD s&s

-regurgitation -chest pain -hoarseness

pancreatitis treatment

-rest pancreas: NPO -opioid analgesics: morphine sulfate -cholecystectomy

irritable bowel syndrome diagnostic test

-serum albumin -ESR -occult stools

peptic ulcer disease

-sore that develops on the lining of the esophagus, stomach, or small intestine -erosion(gradual destruction) of the muscosal lining of the stomach or duodenum= precipitate bleeding and perforation

irritable bowel syndrome risk factors & causes

-stress -caffeine, alcohol -female gender

GERD treatment

-surgery: Linx (strengthens the lower esophageal sphincter) -histamine 2 receptor antagonist: famotidine (pepcid) -avoid: large meals, offending foods(spicy, coffee)

esophageal varices treatment

-surgery: bypass -vasoconstrictor: vasopressin (desmopressin) -nonselective beta blocker: propranolol (inderal)

inflammatory bowel disease treatment

-surgery: colectomy -antidiarrheals: atropine (lomotil) -corticosteroids: prednisone (rayos)

gastritis treatment

-surgery: vagotomy, gastroectomy -antibiotics: amoxicillin -PPIs : pantoprazole -histamine 2 receptor antagonists: famotidine (pepcid) -antiacids: magnesium hydroxide (milk of mag)

peptic ulcer disease treatment

-surgery: vagotomy, gastroectomy -antibiotics: amoxicillin -PPIs : pantoprazole -histamine 2 receptor antagonists: famotidine (pepcid) -antiacids: magnesium hydroxide (milk of mag)

gastritis s&s

-upper abdominal pain (epigastric pain) -dyspepsia -nausea & vomiting

peptic ulcer disease s&s

-upper abdominal pain (epigastric pain) -dyspepsia: heartburn, bloating

pancreatitis s&s

-upper abdominal pain (epigastric pain) -steatorrhea -vomiting

gastritis diagnostic test

-upper endoscopy esophagogastroduodenscopy (EGD) -hgb <12g/dL -c13 urea breath test

hepatitis risk factors & causes

-viral or toxic agent -contaminated food or water -unprotected sex

Lipase

0-110 units/L

Direct (conjugated) bilirubin

0.1-0.3 mg/dL

Total Bilirubin

0.1-1.0 mg/dL

Indirect (unconjugated) bilirubin

0.2-0.8 mg/dL

A client with peptic ulcer disease must begin triple medication therapy. For how long will the client follow this regimen?

10 to 14 days

Morbid obesity is defined as being how many pounds over the person's ideal body weight?

100

The nurse practitioner counsels a patient about being a candidate for bariatric surgery. The patient is 5?4? tall and weighs 300 pounds (IBW is 125 lbs). The nurse tells the patient that, based on the average weight loss after surgery, she should expect to eventually weigh:

120 lbs

Ammonia

15-110 mcg/dL

Moderate depletion of Albumin

2.1-2.7 g/dL

Prealbumin

23-43 mg/dL

Albumin

3.5-5 g/dL

ALP

30-120 units/L or 30-85 IU/L

A health care professional is caring for a patient who is about to begin using dimenhydrinate to prevent motion sickness. Which of the following instructions should the health care professional include when talking with the patient about taking the drug? (Select all that apply.) Sit upright for 30 min after taking the drug. Avoid antacids when taking the drug. Take the drug 30 to 60 min before activities that trigger nausea. Avoid activities that require alertness. Increase fluid and fiber intake.

30-60 min before activity avoid activities require alterness increase fiber/fluids rationale: Sit upright for 30 min after taking the drug is incorrect. Dimenhydrinate, an antihistamine, is unlikely to cause esophagitis, so this precaution is unnecessary. Alendronate (Fosamax), a bisphosphonate that treats osteoporosis, is a drug that requires sitting upright for 30 min after taking it because it can cause esophagitis. Avoid antacids when taking the drug is incorrect. Dimenhydrinate does not interact specifically with antacids. Antacids can decrease the absorption of ranitidine (Zantac), another GI-system drug. Take the drug 30 to 60 min before activities that trigger nausea is correct. The health care professional should tell the patient to take dimenhydrinate 30 to 60 min before activities that cause nausea and again before meals and at bedtime. Avoid activities that require alertness is correct. Dimenhydrinate can cause sedation. The health care professional should suggest avoiding activities that require alertness. Increase fluid and fiber intake is correct. Dimenhydrinate can cause anticholinergic effects, such as dry mouth and constipation. The health care professional should tell the patient to increase activity level and fluid and fiber intake.

A nurse is receiving report from the emergency room regarding a new client being admitted to the medical-surgical unit with a diagnosis of peptic ulcer disease. The nurse expects the age of the client will be between

40 and 60 years

AST

5-40 units/L

Moderate depletion of Prealbumin

5-9 mg/dL

Amylase

56-90 IU/L

When talking with a patient about taking psyllium (Metamucil) to treat constipation, the health care professional should include which of the following instructions? (Select all that apply.) Expect results in 6 to 12 hr. Urinate every 4 hr. Take the drug with at least 8 oz of fluid. Avoid activities that require alertness. Increase fluid and fiber intake.

8 oz of fluids increase fluid and fiber Rationale: Expect results in 6 to 12 hr is incorrect. Psyllium, a fiber supplement, typically results in soft, formed bowel movements 1 to 3 days after beginning therapy. Urinate every 4 hr is incorrect. Psyllium is unlikely to cause urinary retention. Atropine can cause anticholinergic effects, such as urinary retention, and can require scheduled urination to help prevent this effect. Take the drug with at least 8 oz of fluid is correct. To prevent esophageal obstruction, patients should take psyllium with at least 8 oz of fluid, and, after mixing the powdered form, drink it immediately. Avoid activities that require alertness is incorrect. Psyllium is unlikely to cause sedation or dizziness. Prochlorperazine (Compazine) is a GI-system drug that can cause sedation and requires avoiding activities that require alertness. Increase fluid and fiber intake is correct. The health care professional should tell the patient to increase activity and fluid and fiber intake, and to keep track of bowel function.

ALT

8-20 units/L or 3-35 IU/L

constipation

: an abnormal infrequency or irregularity of defecation, abnormal hardening of stools that makes their passage difficult and sometimes painful, a decrease in stool volume, retention of stool in the rectum for a prolonged period often with a sense of incomplete evacuation after defecation, or a persistent sensation of abdominal fullness

diarrhea

: an increased frequency of bowel movements (more than three per day), an increased amount of stool (more than 200 g/day), and altered consistency (i.e., increased liquidity) of stool

fistula

: anatomically abnormal tract that arises between two internal organs or between an internal organ and the body surface

irritable bowel syndrome (IBS)

: chronic functional disorder that affects frequency of defecation and consistency of stool; is associated with no specific structural or biochemical alterations

hemorrhoids

: dilated portions of the anal veins

steatorrhea

: excess of fatty wastes in the feces

inflammatory bowel disease (IBD)

: group of chronic disorders (ulcerative colitis and Crohn's disease) that result in inflammation or ulceration (or both) of the bowel lining

malabsorption

: impaired transport across the mucosa

tenesmus

: ineffective and sometimes painful straining and urge to eliminate either feces or urine

diverticulitis

: inflammation of a diverticulum from obstruction by fecal matter resulting in abscess formation

fecal incontinence

: involuntary passage of feces fissure: normal or abnormal fold, groove, or crack in body tissue

abscess

: localized collection of purulent material surrounded by inflamed tissues

gastrocolic reflex

: peristaltic movements of the large bowel occurring five to six times daily that are triggered by distention of the stomach

diverticulosis

: presence of several diverticula in the intestine

borborygmus

: rumbling noise caused by the movement of gas through the intestines

diverticulum

: saclike out-pouching of the lining of the bowel protruding through the muscle of the intestinal wall

colostomy

: surgical opening into the colon by means of a stoma to allow drainage of bowel contents; one type of fecal diversion

ileostomy

: surgical opening into the ileum by means of a stoma to allow drainage of bowel contents; one type of fecal diversion

Kock pouch

: type of continent ileal reservoir created surgically by making an internal pouch with a portion of the ileum and placing a nipple valve flush with the stoma

Alpha-fetoprotein

<40 mcg/dL

14. A patient was treated in the emergency department and critical care unit after ingesting bleach. What possible complication of the resulting gastritis should the nurse recognize? A) Esophageal or pyloric obstruction related to scarring B) Uncontrolled proliferation of H. pylori C) Gastric hyperacidity related to excessive gastrin secretion D) Chronic referred pain in the lower abdomen

A Feedback: A severe form of acute gastritis is caused by the ingestion of strong acid or alkali, which may cause the mucosa to become gangrenous or to perforate. Scarring can occur, resulting in pyloric stenosis (narrowing or tightening) or obstruction. Chronic referred pain to the lower abdomen is a symptom of peptic ulcer disease, but would not be an expected finding for a patient who has ingested a corrosive substance. Bacterial proliferation and hyperacidity would not occur.

16. A patient is one month postoperative following restrictive bariatric surgery. The patient tells the clinic nurse that he has been having "trouble swallowing" for the past few days. What recommendation should the nurse make? A) Eating more slowly and chewing food more thoroughly B) Taking an OTC antacid or drinking a glass of milk prior to each meal C) Chewing gum to cause relaxation of the lower esophageal sphincter D) Drinking at least 12 ounces of liquid with each meal

A Feedback: Dysphagia may be prevented by educating patients to eat slowly, to chew food thoroughly, and to avoid eating tough foods such as steak or dry chicken or doughy bread. After bariatric procedures, patients should normally not drink beverages with meals. Medications or chewing gum will not alleviate this problem.

12. A patient presents to the walk-in clinic complaining of vomiting and burning in her mid-epigastria. The nurse knows that in the process of confirming peptic ulcer disease, the physician is likely to order a diagnostic test to detect the presence of what? A) Infection with Helicobacter pylori B) Excessive stomach acid secretion C) An incompetent pyloric sphincter D) A metabolic acid-base imbalance

A Feedback: H. pylori infection may be determined by endoscopy and histologic examination of a tissue specimen obtained by biopsy, or a rapid urease test of the biopsy specimen. Excessive stomach acid secretion leads to gastritis; however, peptic ulcers are caused by colonization of the stomach by H. pylori. Sphincter dysfunction and acid-base imbalances do not cause peptic ulcer disease.

2. A patient comes to the clinic complaining of pain in the epigastric region. What assessment question during the health interview would most help the nurse determine if the patient has a peptic ulcer? A) "Does your pain resolve when you have something to eat?" B) "Do over-the-counter pain medications help your pain?" C) "Does your pain get worse if you get up and do some exercise?" D) "Do you find that your pain is worse when you need to have a bowel movement?"

A Feedback: Pain relief after eating is associated with duodenal ulcers. The pain of peptic ulcers is generally unrelated to activity or bowel function and may or may not respond to analgesics.

40. A patient has come to the clinic complaining of pain just above her umbilicus. When assessing the patient, the nurse notes Sister Mary Joseph's nodules. The nurse should refer the patient to the primary care provider to be assessed for what health problem? A) A GI malignancy B) Dumping syndrome C) Peptic ulcer disease D) Esophageal/gastric obstruction

A Feedback: Palpable nodules around the umbilicus, called Sister Mary Joseph's nodules, are a sign of a GI malignancy, usually a gastric cancer. This would not be a sign of dumping syndrome, peptic ulcer disease, or esophageal/gastric obstruction.

38. A patient with gastritis required hospital treatment for an exacerbation of symptoms and receives a subsequent diagnosis of pernicious anemia due to malabsorption. When planning the patient's continuing care in the home setting, what assessment question is most relevant? A) "Does anyone in your family have experience at giving injections?" B) "Are you going to be anywhere with strong sunlight in the next few months?" C) "Are you aware of your blood type?" D) "Do any of your family members have training in first aid?"

A Feedback: Patients with malabsorption of vitamin B12 need information about lifelong vitamin B12 injections; the nurse may instruct a family member or caregiver how to administer the injections or make arrangements for the patient to receive the injections from a health care provider. Questions addressing sun exposure, blood type and first aid are not directly relevant.

28. Diagnostic imaging and physical assessment have revealed that a patient with peptic ulcer disease has suffered a perforated ulcer. The nurse recognizes that emergency interventions must be performed as soon as possible in order to prevent the development of what complication? A) Peritonitis B) Gastritis C) Gastroesophageal reflux D) Acute pancreatitis

A Feedback: Perforation is the erosion of the ulcer through the gastric serosa into the peritoneal cavity without warning. Chemical peritonitis develops within a few hours of perforation and is followed by bacterial peritonitis. Gastritis, reflux, and pancreatitis are not acute complications of a perforated ulcer.

8. A nurse in the postanesthesia care unit admits a patient following resection of a gastric tumor. Following immediate recovery, the patient should be placed in which position to facilitate patient comfort and gastric emptying? A) Fowler's B) Supine C) Left lateral D) Left Sim's

A Feedback: Positioning the patient in a Fowler's position postoperatively promotes comfort and facilitates emptying of the stomach following gastric surgery. Any position that involves lying down delays stomach emptying and is not recommended for this type of patient. Supine positioning and the left lateral (left Sim's) position do not achieve this goal.

3. A patient with a diagnosis of peptic ulcer disease has just been prescribed omeprazole (Prilosec). How should the nurse best describe this medication's therapeutic action? A) "This medication will reduce the amount of acid secreted in your stomach." B) "This medication will make the lining of your stomach more resistant to damage." C) "This medication will specifically address the pain that accompanies peptic ulcer disease." D) "This medication will help your stomach lining to repair itself."

A Feedback: Proton pump inhibitors like Prilosec inhibit the synthesis of stomach acid. PPIs do not increase the durability of the stomach lining, relieve pain, or stimulate tissue repair.

29. A nurse is performing the admission assessment of a patient whose high body mass index (BMI) corresponds to class III obesity. In order to ensure empathic and patient-centered care, the nurse should do which of the following? A) Examine one's own attitudes towards obesity in general and the patient in particular. B) Dialogue with the patient about the lifestyle and psychosocial factors that resulted in obesity. C) Describe one's own struggles with weight gain and weight loss to the patient. D) Elicit the patient's short-term and long-term goals for weight loss.

A Feedback: Studies suggest that health care providers, including nurses, harbor negative attitudes towards obese patients. Nurses have a responsibility to examine these attitudes and change them accordingly. This is foundational to all other areas of assessing this patient.

11. A patient who experienced an upper GI bleed due to gastritis has had the bleeding controlled and the patient's condition is now stable. For the next several hours, the nurse caring for this patient should assess for what signs and symptoms of recurrence? A) Tachycardia, hypotension, and tachypnea B) Tarry, foul-smelling stools C) Diaphoresis and sudden onset of abdominal pain D) Sudden thirst, unrelieved by oral fluid administration

A Feedback: Tachycardia, hypotension, and tachypnea are signs of recurrent bleeding. Patients who have had one GI bleed are at risk for recurrence. Tarry stools are expected short-term findings after a hemorrhage. Hemorrhage is not normally associated with sudden thirst or diaphoresis.

35. A patient has received a diagnosis of gastric cancer and is awaiting a surgical date. During the preoperative period, the patient should adopt what dietary guidelines? A) Eat small, frequent meals with high calorie and vitamin content. B) Eat frequent meals with an equal balance of fat, carbohydrates, and protein. C) Eat frequent, low-fat meals with high protein content. D) Try to maintain the pre-diagnosis pattern of eating.

A Feedback: The nurse encourages the patient to eat small, frequent portions of nonirritating foods to decrease gastric irritation. Food supplements should be high in calories, as well as vitamins A and C and iron, to enhance tissue repair.

Which of the following clients is at highest risk for peptic ulcer disease?

A 52-year-old male accountant

A client is suspected of having cirrhosis of the liver. What diagnostic procedure will the nurse prepare the client for in order to obtain a confirmed diagnosis? a) Platelet count b) A liver biopsy c) A CT scan d) A prothrombin time

A liver biopsy A liver biopsy, which reveals hepatic fibrosis, is the most conclusive diagnostic procedure. It can be performed in the radiology department with ultrasound or CT to identify appropriate placement of the trocar or biopsy needle. A prothrombin time and platelet count will assist with determining if the client is at increased risk for bleeding.

A patient is scheduled for a Billroth I procedure for ulcer management. What does the nurse understand will occur when this procedure is performed?

A partial gastrectomy is performed with anastomosis of the stomach segment to the duodenum.

nurses is caring for a client who is 4hr post-op following a laparoscopic cholecystectomy. which of the following findings should the nurse expect? A) right shoulder pain B) urine output 20ml/hr C) temp of 101.1 D) O2 stat 92%

A) right shoulder pain -due to gas (carbon dioxide) injected into abdominal cavity during laparoscopic procedure

nurse is caring for pt with celiac disease. which of the following foods should the nurse remove from clients tray? A) wheat toast B) tapioca pudding C) hard boiled eggs D) mashed potatoes

A) wheat toast -celiac disease is an autoimmune disease characterized by intolerance to wheat, barley, and rye.

39. A nurse is presenting a class at a bariatric clinic about the different types of surgical procedures offered by the clinic. When describing the implications of different types of surgeries, the nurse should address which of the following topics? Select all that apply. A) Specific lifestyle changes associated with each procedure B) Implications of each procedure for eating habits C) Effects of different surgeries on bowel function D) Effects of various bariatric surgeries on fertility E) Effects of different surgeries on safety of future immunizations

A, B, C Feedback: Different bariatric surgical procedures entail different lifestyle modifications; patients must be well informed about the specific lifestyle changes, eating habits, and bowel habits that may result from a particular procedure. Bariatric surgeries do not influence the future use of immunizations or fertility, though pregnancy should be avoided for 18 months after bariatric surgery.

GERD and new prescription for Metoclopramide. monitor for what side effect

ATAXIA

Which of the following is an outcome of histamine 2 (H2)-receptor antagonists blocking the action of histamine in the stomach?

Acid secretion is reduced.

A client reports to the clinic, stating that she rapidly developed headache, abdominal pain, nausea, hiccuping, and fatigue about 2 hours ago. For dinner, she ate buffalo chicken wings and beer. Which of the following medical conditions is most consistent with the client's presenting problems?

Acute gastritis

Which of the following terms is used to describe a chronic liver disease in which scar tissue surrounds the portal areas? a) Postnecrotic cirrhosis b) Compensated cirrhosis c) Alcoholic cirrhosis d) Biliary cirrhosis

Alcoholic cirrhosis This type of cirrhosis is due to chronic alcoholism and is the most common type of cirrhosis. In postnecrotic cirrhosis, there are broad bands of scar tissue, which are a late result of a previous acute viral hepatitis. In biliary cirrhosis, scarring occurs in the liver around the bile ducts. Compensated cirrhosis is a general term given to the state of liver disease in which the liver continues to be able to function effectively.

Which of the following drugs has protocols that require patients to meet specific risk-management criteria and sign a treatment agreement before the health care professional can administer the drug? Lubiprostone (Amitiza) Azathioprine (Imuran) Sulfasalazine (Azulfidine) Alosetron (Lotronex)

Alosetron (Lotronex) rationale: Patients who take alosetron, a serotonin 5-HT3 receptor antagonist, can develop severe constipation that can lead to impaction, bowel obstruction, perforation, and potentially fatal ischemic colitis. Because of these risks, health care professionals must inform patients of the benefits and risks of the drug therapy, and patients must sign a treatment agreement.

Lactulose (Cephulac) is administered to a patient diagnosed with hepatic encephalopathy to reduce which of the following? a) Calcium b) Alcohol c) Bicarbonate d) Ammonia

Ammonia Lactulose (Cephulac) is administered to reduce serum ammonia levels. Cephulac does not influence calcium, bicarbonate, or alcohol levels.

Which of the following is a manifestation of decreased hemoglobin?

Anemia

22. A nurse is providing care for a patient who has a diagnosis of irritable bowel syndrome (IBS). When planning this patient's care, the nurse should collaborate with the patient and prioritize what goal? A) Patient will accurately identify foods that trigger symptoms. B) Patient will demonstrate appropriate care of his ileostomy. C) Patient will demonstrate appropriate use of standard infection control precautions. D) Patient will adhere to recommended guidelines for mobility and activity.

Ans: A Feedback: A major focus of nursing care for the patient with IBS is to identify factors that exacerbate symptoms. Surgery is not used to treat this health problem and infection control is not a concern that is specific to this diagnosis. Establishing causation likely is more important to the patient than managing physical activity.

17. A nurse is caring for a patient admitted with symptoms of an anorectal infection, cultures indicate that the patient has a viral infection. The nurse should anticipate the administration of what drug? A) Acyclovir (Zovirax) B) Doxycycline (Vibramycin) C) Penicillin (penicillin D) Metronidazole (Flagyl)

Ans: A Feedback: Acyclovir (Zovirax) is often given to patients with viral anorectal infections. Doxycycline (Vibramycin) and penicillin (penicillin G) are drugs of choice for bacterial infections. Metronidazole (Flagyl) is used for other infections with a bacterial etiology it is ineffective against viruses.

5. A 35-year-old male patient presents at the emergency department with symptoms of a small bowel obstruction. In collaboration with the primary care provider, what intervention should the nurse prioritize? A) Insertion of a nasogastric tube B) Insertion of a central venous catheter C) Administration of a mineral oil enema D) Administration of a glycerin suppository and an oral laxative

Ans: A Feedback: Decompression of the bowel through a nasogastric tube is necessary for all patients with small bowel obstruction. Peripheral IV access is normally sufficient. Enemas, suppositories, and laxatives are not indicated if an obstruction is present.

38. A teenage patient with a pilonidal cyst has been brought for care by her mother. The nurse who is contributing to the patient's care knows that treatment will be chosen based on what risk? A) Risk for infection B) Risk for bowel incontinence C) Risk for constipation D) Risk for impaired tissue perfusion

Ans: A Feedback: Pilonidal cysts frequently develop into an abscess, necessitating surgical repair. These cysts do not contribute to bowel incontinence, constipation, or impaired tissue perfusion.

8. A nurse is presenting an educational event to a local community group. When speaking about colorectal cancer, what risk factor should the nurse cite? A) High levels of alcohol consumption B) History of bowel obstruction C) History of diverticulitis D) Longstanding psychosocial stress

Ans: A Feedback: Risk factors include high alcohol intake cigarette smoking, and high-fact, high-protein, low-fiber diet. Diverticulitis, obstruction, and stress are not noted as risk factors for colorectal cancer.

30. The nurse is providing care for a patient whose inflammatory bowel disease has necessitated hospital treatment. Which of the following would most likely be included in the patient's medication regimen? A) Anticholinergic medications 30 minutes before a meal B) Antiemetics on a PRN basis C) Vitamin B12 injections to prevent pernicious anemia D) Beta adrenergic blockers to reduce bowel motility

Ans: A Feedback: The nurse administers anticholinergic medications 30 minutes before a meal as prescribed to decrease intestinal motility and administers analgesics as prescribed for pain. Antiemetics, vitamin B12 injections and beta blockers do not address the signs, symptoms, or etiology of inflammatory bowel disease.

2. A nurse is preparing to provide care for a patient whose exacerbation of ulcerative colitis has required hospital admission. During an exacerbation of this health problem, the nurse would anticipate that the patient's stools will have what characteristics? A) Watery with blood and mucus B) Hard and black or tarry C) Dry and streaked with blood D) Loose with visible fatty streaks

Ans: A Feedback: The predominant symptoms of ulcerative colitis are diarrhea and abdominal pain. Stools may be bloody and contain mucus. Stools are not hard, dry, tarry, black or fatty in patients who have ulcerative colitis.

7. A patient is admitted to the medical unit with a diagnosis of intestinal obstruction. When planning this patient's care, which of the following nursing diagnoses should the nurse prioritize? A) Ineffective Tissue Perfusion Related to Bowel Ischemia B) Imbalanced Nutrition: Less Than Body Requirements Related to Impaired Absorption C) Anxiety Related to Bowel Obstruction and Subsequent Hospitalization D) Impaired Skin Integrity Related to Bowel Obstruction

Ans: A Feedback: When the bowel is completely obstructed, the possibility of strangulation and tissue necrosis (i.e., tissue death) warrants surgical intervention. As such, this immediate physiologic need is a nursing priority. Nutritional support and management of anxiety are necessary, but bowel ischemia is a more immediate threat. Skin integrity is not threatened.

29. A nurse is caring for a patient who has been admitted to the hospital with diverticulitis. Which of the following would be appropriate nursing diagnoses for this patient? Select all that apply. A) Acute Pain Related to Increased Peristalsis and GI Inflammation B) Activity Intolerance Related to Generalized Weakness C) Bowel Incontinence Related to Increased Intestinal Peristalsis D) Deficient Fluid Volume Related to Anorexia, Nausea, and Diarrhea E) Impaired Urinary Elimination Related to GI Pressure on the Bladder

Ans: A, B, D Feedback: Patients with diverticulitis are likely to experience pain and decreased activity levels, and are at risk of fluid volume deficit. The patient is unlikely to experience fecal incontinence and urinary function is not directly influenced.

37. A patient with a diagnosis of colon cancer is 2 days postoperative following bowel resection and anastomosis. The nurse has planned the patient's care in the knowledge of potential complications. What assessment should the nurse prioritize? A) Close monitoring of temperature B) Frequent abdominal auscultation C) Assessment of hemoglobin, hematocrit, and red blood cell levels D) Palpation of peripheral pulses and leg girth

Ans: B Feedback: After bowel surgery, it is important to frequently assess the abdomen, including bowel sounds and abdominal girth, to detect bowel obstruction. The resumption of bowel motility is a priority over each of the other listed assessments, even though each should be performed by the nurse.

18. A nurse caring for a patient with colorectal cancer is preparing the patient for upcoming surgery. The nurse administers cephalexin (Keflex) to the patient and explains what rationale? A) To treat any undiagnosed infections B) To reduce intestinal bacteria levels C) To reduce bowel motility D) To reduce abdominal distention postoperatively

Ans: B Feedback: Antibiotics such a kanamycin (Kantrex), neomycin (Mycifradin), and cephalexin (Keflex) are administered orally the day before surgery to reduce intestinal bacterial. Preoperative antibiotics are not given to treat undiagnosed infections, reduce motility, or prevent abdominal distention.

28. A nurse is providing care for a patient whose recent colostomy has contributed to a nursing diagnosis of Disturbed Body Image Related to Colostomy. What intervention best addresses this diagnosis? A) Encourage the patient to conduct online research into colostomies. B) Engage the patient in the care of the ostomy to the extent that the patient is willing. C) Emphasize the fact that the colostomy was needed to alleviate a much more serious health problem. D) Emphasize the fact that the colostomy is temporary measure and is not permanent.

Ans: B Feedback: For some patients, becoming involved in the care of the ostomy helps to normalize it and enhance familiarity. Emphasizing the benefits of the intervention is unlikely to improve the patient's body image, since the benefits are likely already known. Online research is not likely to enhance the patient's body image and some ostomies are permanent.

3. A patient has had an ileostomy created for the treatment of irritable bowel disease and the patient is now preparing for discharge. What should the patient be taught about changing this device in the home setting? A) Apply antibiotic ointment as ordered after cleaning the stoma. B) Apply a skin barrier to the peristomal skin prior to applying the pouch. C) Dispose of the clamp with each bag change. D) Cleanse the area surrounding the stoma with alcohol or chlorhexidine.

Ans: B Feedback: Guidelines for changing an ileostomy appliance are as follows. Skin should be washed with soap and water, and dried. A skin barrier should be applied to the peristomal skin prior to applying the pouch. Clamps are supplied one per box and should be reused with each bag change. Topical antibiotics are not utilized, but an antifungal spray or powder may be used.

27. During a patient's scheduled home visit, an older adult patient has stated to the community health nurse that she has been experiencing hemorrhoids of increasing severity in recent months. The nurse should recommend which of the following? A) Regular application of an OTC antibiotic ointment B) Increased fluid and fiber intake C) Daily use of OTC glycerin suppositories D) Use of an NSAID to reduce inflammation

Ans: B Feedback: Hemorrhoid symptoms and discomfort can be relieved by good personal hygiene and by avoiding excessive straining during defecation. A high-residue diet that contains fruit and bran along with an increased fluid intake may be all the treatment that is necessary to promote the passage of soft, bulky stools to prevent straining. Antibiotics, regular use of suppositories, and NSAIDs are not recommended, as they do not address the etiology of the health problem.

10. A nursing instructor is discussing hemorrhoids with the nursing class. Which patients would the nursing instructor identify as most likely to develop hemorrhoids? A) A 45-year-old teacher who stands for 6 hours per day B) A pregnant woman at 28 weeks' gestation C) A 37-year-old construction worker who does heavy lifting D) A 60-year-old professional who is under stress

Ans: B Feedback: Hemorrhoids commonly affect 50% of patients after the age of 50. Pregnancy may initiate hemorrhoids or aggravate existing ones. This is due to increased constipation during pregnancy. The significance of pregnancy is greater than that of standing, lifting, or stress in the development of hemorrhoids.

20. A nurse caring for a patient with a newly created ileostomy assesses the patient and notes that the patient has had not ostomy output for the past 12 hours. The patient also complains of worsening nausea. What is the nurse's priority action? A) Facilitate a referral to the wound-ostomy-continence (WOC) nurse. B) Report signs and symptoms of obstruction to the physician. C) Encourage the patient to mobilize in order to enhance motility. D) Contact the physician and obtain a swab of the stoma for culture.

Ans: B Feedback: It is important to report nausea and abdominal distention, which may indicate intestinal obstruction. This requires prompt medical intervention. Referral to the WOC nurse is not an appropriate short-term response, since medical treatment is necessary. Physical mobility will not normally resolve an obstruction. There is no need to collect a culture from the stoma, because infection is unrelated to this problem.

15. A nurse is caring for a patient with constipation whose primary care provider has recommended senna (Senokot) for the management of this condition. The nurse should provide which of the following education points? A) "Limit your fluid intake temporarily so you don't get diarrhea." B) "Avoid taking the drug on a long-term basis." C) "Make sure to take a multivitamin with each dose." D) "Take this on an empty stomach to ensure maximum effect."

Ans: B Feedback: Laxatives should not be taken on an ongoing basis in order to reduce the risk of dependence. Fluid should be increased, not limited, and there is no need to take each dose with a multivitamin. Senna does not need to be taken on an empty stomach.

6. A patient admitted with inflammatory bowel disease asks the nurse for help with menu selections. What menu selection is most likely the best choice for this patient? A) Spinach B) Tofu C) Multigrain bagel D) Blueberries

Ans: B Feedback: Nutritional management of inflammatory bowel disease requires ingestion of a diet that is bland, low-residue, high-protein, and high-vitamin. Tofu meets each of the criteria. Spinach, multigrain bagels, and blueberries are not low-residue.

39. A nurse at an outpatient surgery center is caring for a patient who had a hemorrhoidectomy. What discharge education topics should the nurse address with this patient? A) The appropriate use of antibiotics to prevent postoperative infection B) The correct procedure for taking a sitz bath C) The need to eat a low-residue, low-fat diet for the next 2 weeks D) The correct technique for keeping the perianal region clean without the use of water

Ans: B Feedback: Sitz baths are usually indicated after perianal surgery. A low-residue, low-fat diet is not necessary and water is used to keep the region clean. Postoperative antibiotics are not normally prescribed.

16. The nurse is caring for a patient who is undergoing diagnostic testing for suspected malabsorption. When taking this patient's health history and performing the physical assessment, the nurse should recognize what finding as most consistent with this diagnosis? A) Recurrent constipation coupled with weight loss B) Foul-smelling diarrhea that contains fat C) Fever accompanied by a rigid, tender abdomen D) Bloody bowel movements accompanied by fecal incontinence

Ans: B Feedback: The hallmarks of malabsorption syndrome from any cause are diarrhea or frequent, loose, bulky, foul-smelling stools that have increased fat content and are often grayish (steatorrhea). Constipation and bloody bowel movements are not suggestive of malabsorption syndromes. Fever and a tender, rigid abdomen are associated with peritonitis.

12. An older adult who resides in an assisted living facility has sought care from the nurse because of recurrent episodes of constipation. Which of the following actions should the nurse first perform? A) Encourage the patient to take stool softener daily. B) Assess the patient's food and fluid intake. C) Assess the patient's surgical history. D) Encourage the patient to take fiber supplements.

Ans: B Feedback: The nurse should follow the nursing process and perform an assessment prior to interventions. The patient's food and fluid intake is more likely to affect bowel function than surgery.

13. A 16-year-old presents at the emergency department complaining of right lower quadrant pain and is subsequently diagnosed with appendicitis. When planning this patient's nursing care, the nurse should prioritize what nursing diagnosis? A) Imbalanced Nutrition: Less Than Body Requirements Related to Decreased Oral Intake B) Risk for Infection Related to Possible Rupture of Appendix C) Constipation Related to Decreased Bowel Motility and Decreased Fluid Intake D) Chronic Pain Related to Appendicitis

Ans: B Feedback: The patient with a diagnosis of appendicitis has an acute risk of infection related to the possibility of rupture. This immediate physiologic risk is a priority over nutrition and constipation, though each of these concerns should be addressed by the nurse. The pain associated with appendicitis is acute, not chronic.

4. A patient admitted with acute diverticulitis has experienced a sudden increase in temperature and complains of a sudden onset of exquisite abdominal tenderness. The nurse's rapid assessment reveals that the patient's abdomen is uncharacteristically rigid on palpation. What is the nurse's best response? A) Administer a Fleet enema as ordered and remain with the patient. B) Contact the primary care provider promptly and report these signs of perforation. C) Position the patient supine and insert an NG tube. D) Page the primary care provider and report that the patient may be obstructed.

Ans: B Feedback: The patient's change in status is suggestive of perforation, which is a surgical emergency. Obstruction does not have this presentation involving fever and abdominal rigidity. An enema would be strongly contraindicated. An order is needed for NG insertion and repositioning is not a priority.

25. An adult patient has been diagnosed with diverticular disease after ongoing challenges with constipation. The patient will be treated on an outpatient basis. What components of treatment should the nurse anticipate? Select all that apply. A) Anticholinergic medications B) Increased fiber intake C) Enemas on alternating days D) Reduced fat intake E) Fluid reduction

Ans: B, D Feedback: Patients whose diverticular disease does not warrant hospital treatment often benefit from a high-fiber, low-fat diet. Neither enemas nor anticholinergics are indicated, and fluid intake is encouraged.

33. A nurse is assessing a patient's stoma on postoperative day 3. The nurse notes that the stoma has a shiny appearance and a bright red color. How should the nurse best respond to this assessment finding? A) Irrigate the ostomy to clear a possible obstruction. B) Contact the primary care provider to report this finding. C) Document that the stoma appears healthy and well perfused. D) Document a nursing diagnosis of Impaired Skin Integrity.

Ans: C Feedback: A healthy, viable stoma should be shiny and pink to bright red. This finding does not indicate that the stoma is blocked or that skin integrity is compromised.

9. A patient's screening colonoscopy revealed the presence of numerous polyps in the large bowel. What principle should guide the subsequent treatment of this patient's health problem? A) Adherence to a high-fiber diet will help the polyps resolve. B) The patient should be assured that these are a normal, age-related physiologic change. C) The patient's polyps constitute a risk factor for cancer. D) The presence of polyps is associated with an increased risk of bowel obstruction.

Ans: C Feedback: Although most polyps do not develop into invasive neoplasms, they must be identified and followed closely. They are very common, but are not classified as a normal, age-related physiologic change. Diet will not help them resolve and they do not typically lead to obstructions.

24. An older adult has a diagnosis of Alzheimer's disease and has recently been experiencing fecal incontinence. However, the nurse has observed no recent change in the character of the patient's stools. What is the nurse's most appropriate intervention? A) Keep a food diary to determine the foods that exacerbate the patient's symptoms. B) Provide the patient with a bland, low-residue diet. C) Toilet the patient on a frequent, scheduled basis. D) Liaise with the primary care provider to obtain an order for loperamide.

Ans: C Feedback: Because the patient's fecal incontinence is most likely attributable to cognitive decline, frequent toileting is an appropriate intervention. Loperamide is unnecessary in the absence of diarrhea. Specific foods are not likely to be a cause of, or solution to, this patient's health problem.

26. A patient's health history is suggestive of inflammatory bowel disease. Which of the following would suggest Crohn's disease, rather that ulcerative colitis, as the cause of the patient's signs and symptoms? A) A pattern of distinct exacerbations and remissions B) Severe diarrhea C) An absence of blood in stool D) Involvement of the rectal mucosa

Ans: C Feedback: Bloody stool is far more common in cases of UC than in Crohn's. Rectal involvement is nearly 100% in cases of UC (versus 20% in Crohn's) and patients with UC typically experience severe diarrhea. UC is also characterized by a pattern of remissions and exacerbations, while Crohn's often has a more prolonged and variable course.

11. A nurse is planning discharge teaching for a 21-year-old patient with a new diagnosis of ulcerative colitis. When planning family assessment, the nurse should recognize that which of the following factors will likely have the greatest impact on the patient's coping after discharge? A) The family's ability to take care of the patient's special diet needs B) The family's ability to monitor the patient's changing health status C) The family's ability to provide emotional support D) The family's ability to manage the patient's medication regimen

Ans: C Feedback: Emotional support from the family is key to the patient's coping after discharge. A 21-year-old would be expected to self-manage the prescribed medication regimen and the family would not be primarily responsible for monitoring the patient's health status. It is highly beneficial if the family is willing and able to accommodate the patient's dietary needs, but emotional support is paramount and cannot be solely provided by the patient alone.

31. A patient's colorectal cancer has necessitated a hemicolectomy with the creation of a colostomy. In the 4 days since the surgery, the patient has been unwilling to look at the ostomy or participate in any aspects of ostomy care. What is the nurse's most appropriate response to this observation? A) Ensure that the patient knows that he or she will be responsible for care after discharge. B) Reassure the patient that many people are fearful after the creation of an ostomy. C) Acknowledge the patient's reluctance and initiate discussion of the factors underlying it. D) Arrange for the patient to be seen by a social worker or spiritual advisor.

Ans: C Feedback: If the patient is reluctant to participate in ostomy care, the nurse should attempt to dialogue about this with the patient and explore the factors that underlie it. It is presumptive to assume that the patient's behavior is motivated by fear. Assessment must precede referrals and emphasizing the patient's responsibilities may or may not motivate the patient.

21. A nurse is working with a patient who is learning to care for a continent ileostomy (Kock pouch). Following the initial period of healing, the nurse is teaching the patient how to independently empty the ileostomy. The nurse should teach the patient to do which of the following actions? A) Aim to eventually empty the pouch every 90 minutes. B) Avoid emptying the pouch until it is visibly full. C) Insert the catheter approximately 5 cm into the pouch. D) Aspirate the contents of the pouch using a 60 mL piston syringe.

Ans: C Feedback: To empty a Kock pouch, the catheter is gently inserted approximately 5 cm to the point of the valve or nipple. The length of time between drainage periods is gradually increased until the reservoir needs to be drained only every 4 to 6 hours and irrigated once each day. It is not appropriate to wait until the pouch is full, and this would not be visible. The contents of the pouch are not aspirated.

14. A nurse is talking with a patient who is scheduled to have a hemicolectomy with the creation of a colostomy. The patient admits to being anxious, and has many questions concerning the surgery, the care of a stoma, and necessary lifestyle changes. Which of the following nursing actions is most appropriate? A) Reassure the patient that the procedure is relatively low risk and that patients are usually successful in adjusting to an ostomy. B) Provide the patient with educational materials that match the patient's learning style. C) Encourage the patient to write down these concerns and questions to bring forward to the surgeon. D) Maintain an open dialogue with the patient and facilitate a referral to the wound-ostomy-continence (WOC) nurse.

Ans: D Feedback: A wound-ostomy-continence (WOC) nurse is a registered nurse who has received advanced education in an accredited program to care for patients with stomas. The enterostomal nurse therapist can assist with the selection of an appropriate stoma site, teach about stoma care, and provide emotional support. The surgeon is less likely to address the patient's psychosocial and learning needs. Reassurance does not address the patient's questions, and education may or may not alleviate anxiety.

34. A patient has been diagnosed with a small bowel obstruction and has been admitted to the medical unit. The nurse's care should prioritize which of the following outcomes? A) Preventing infection B) Maintaining skin and tissue integrity C) Preventing nausea and vomiting D) Maintaining fluid and electrolyte balance

Ans: D Feedback: All of the listed focuses of care are important for the patient with a small bowel obstruction. However, the patient's risk of fluid and electrolyte imbalances is an immediate threat to safety, and is a priority in nursing assessment and interventions.

32. A nurse is caring for an older adult who has been experiencing severe Clostridium difficile-related diarrhea. When reviewing the patient's most recent laboratory tests, the nurse should prioritize which of the following? A) White blood cell level B) Creatinine level C) Hemoglobin level D) Potassium level

Ans: D Feedback: In elderly patients, it is important to monitor the patient's serum electrolyte levels closely. Diarrhea is less likely to cause an alteration in white blood cell, creatinine, and hemoglobin levels.

23. A patient has been experiencing disconcerting GI symptoms that have been worsening in severity. Following medical assessment, the patient has been diagnosed with lactose intolerance. The nurse should recognize an increased need for what form of health promotion? A) Annual screening colonoscopies B) Adherence to recommended immunization schedules C) Regular blood pressure monitoring D) Frequent screening for osteoporosis

Ans: D Feedback: Persons with lactose intolerance often experience hypocalcemia and a consequent risk of osteoporosis related to malabsorption of calcium. Lactose intolerance does not create an increased need for screening for colorectal cancer, immunizations, or blood pressure monitoring.

36. A patient has been experiencing occasional episodes of constipation and has been unable to achieve consistent relief by increasing physical activity and improving his diet. What pharmacologic intervention should the nurse recommend to the patient for ongoing use? A) Mineral oil enemas B) Bisacodyl (Dulcolax) C) Senna (Senokot) D) Psyllium hydrophilic mucilloid (Metamucil)

Ans: D Feedback: Psyllium hydrophilic mucilloid (Metamucil) is a bulk-forming laxative that is safe for ongoing use. None of the other listed laxatives should be used on an ongoing basis because of the risk of dependence.

19. A nurse is teaching a group of adults about screening and prevention of colorectal cancer. The nurse should describe which of the following as the most common sign of possible colon cancer? A) Development of new hemorrhoids B) Abdominal bloating and flank pain C) Unexplained weight gain D) Change in bowel habits

Ans: D Feedback: The most common presenting symptom associated with colorectal cancer is a change in bowel habits. The passage of blood is the second most common symptom. Symptoms may also include unexplained anemia, anorexia, weight loss, and fatigue. Hemorrhoids and bloating are atypical.

40. Which of the following is the most plausible nursing diagnosis for a patient whose treatment for colon cancer has necessitated a colonostomy? A) Risk for Unstable Blood Glucose Due to Changes in Digestion and Absorption B) Unilateral Neglect Related to Decreased Physical Mobility C) Risk for Excess Fluid Volume Related to Dietary Changes and Changes In Absorption D) Ineffective Sexuality Patterns Related to Changes in Self-Concept

Ans: D Feedback: The presence of an ostomy frequently has an effect on sexuality, this should be addressed thoughtfully in nursing care. None of the other listed diagnoses reflects the physiologic changes that result from colorectal surgery.

35. A patient's large bowel obstruction has failed to resolve spontaneously and the patient's worsening condition has warranted admission to the medical unit. Which of the following aspects of nursing care is most appropriate for this patient? A) Administering bowel stimulants as ordered B) Administering bulk-forming laxatives as ordered C) Performing deep palpation as ordered to promote peristalsis D) Preparing the patient for surgical bowel resection

Ans: D Feedback: The usual treatment for a large bowel obstruction is surgical resection to remove the obstructing lesion. Administration of laxatives or bowel stimulants are contraindicated if the bowel is obstructed. Palpation would be painful and has no therapeutic benefit.

27. A patient has a recent diagnosis of chronic pancreatitis and is undergoing diagnostic testing to determine pancreatic islet cell function. The nurse should anticipate what diagnostic test? A) Glucose tolerance test B) ERCP C) Pancreatic biopsy D) Abdominal ultrasonography

Ans: A Feedback: A glucose tolerance test evaluates pancreatic islet cell function and provides necessary information for making decisions about surgical resection of the pancreas. This specific clinical information is not provided by ERCP, biopsy, or ultrasound.

7. A patient has been scheduled for an ultrasound of the gallbladder the following morning. What should the nurse do in preparation for this diagnostic study? A) Have the patient refrain from food and fluids after midnight. B) Administer the contrast agent orally 10 to 12 hours before the study. C) Administer the radioactive agent intravenously the evening before the study. D) Encourage the intake of 64 ounces of water 8 hours before the study.

Ans: A Feedback: An ultrasound of the gallbladder is most accurate if the patient fasts overnight, so that the gallbladder is distended. Contrast and radioactive agents are not used when performing ultrasonography of the gallbladder, as an ultrasound is based on reflected sound waves.

20. During a health education session, a participant has asked about the hepatitis E virus. What prevention measure should the nurse recommend for preventing infection with this virus? A) Following proper hand-washing techniques B) Avoiding chemicals that are toxic to the liver C) Wearing a condom during sexual contact D) Limiting alcohol intake

Ans: A Feedback: Avoiding contact with the hepatitis E virus through good hygiene, including hand-washing, is the major method of prevention. Hepatitis E is transmitted by the fecal-oral route, principally through contaminated water in areas with poor sanitation. Consequently, none of the other listed preventative measures is indicated.

5. A nurse is caring for a patient with cancer of the liver whose condition has required the insertion of a percutaneous biliary drainage system. The nurse's most recent assessment reveals the presence of dark green fluid in the collection container. What is the nurse's best response to this assessment finding? A) Document the presence of normal bile output. B) Irrigate the drainage system with normal saline as ordered. C) Aspirate a sample of the drainage for culture. D) Promptly report this assessment finding to the primary care provider.

Ans: A Feedback: Bile is usually a dark green or brownish-yellow color, so this would constitute an expected assessment finding, with no other action necessary.

24. An adult patient has been admitted to the medical unit for the treatment of acute pancreatitis. What nursing action should be included in this patient's plan of care? A) Measure the patient's abdominal girth daily. B) Limit the use of opioid analgesics. C) Monitor the patient for signs of dysphagia. D) Encourage activity as tolerated.

Ans: A Feedback: Due to the risk of ascites, the nurse should monitor the patient's abdominal girth. There is no specific need to avoid the use of opioids or to monitor for dysphagia, and activity is usually limited.

29. A group of nurses have attended an inservice on the prevention of occupationally acquired diseases that affect healthcare providers. What action has the greatest potential to reduce a nurse's risk of acquiring hepatitis C in the workplace? A) Disposing of sharps appropriately and not recapping needles B) Performing meticulous hand hygiene at the appropriate moments in care C) Adhering to the recommended schedule of immunizations D) Wearing an N95 mask when providing care for patients on airborne precautions

Ans: A Feedback: HCV is bloodborne. Consequently, prevention of needlestick injuries is paramount. Hand hygiene, immunizations and appropriate use of masks are important aspects of overall infection control, but these actions do not directly mitigate the risk of HCV.

4. A patient's abdominal ultrasound indicates cholelithiasis. When the nurse is reviewing the patient's laboratory studies, what finding is most closely associated with this diagnosis? A) Increased bilirubin B) Decreased serum cholesterol C) Increased blood urea nitrogen (BUN) D) Decreased serum alkaline phosphatase level

Ans: A Feedback: If the flow of blood is impeded, bilirubin, a pigment derived from the breakdown of red blood cells, does not enter the intestines. As a result, bilirubin levels in the blood increase. Cholesterol, BUN, and alkaline phosphatase levels are not typically affected.

15. A patient with a liver mass is undergoing a percutaneous liver biopsy. What action should the nurse perform when assisting with this procedure? A) Position the patient on the right side with a pillow under the costal margin after the procedure. B) Administer 1 unit of albumin 90 minutes before the procedure as ordered. C) Administer at least 1 unit of packed red blood cells as ordered the day before the scheduled procedure. D) Confirm that the patient's electrolyte levels have been assessed prior to the procedure.

Ans: A Feedback: Immediately after a percutaneous liver biopsy, assist the patient to turn onto the right side and place a pillow under the costal margin. Prior administration of albumin or PRBCs is unnecessary. Coagulation tests should be performed, but electrolyte analysis is not necessary.

38. A nurse on a solid organ transplant unit is planning the care of a patient who will soon be admitted upon immediate recovery following liver transplantation. What aspect of nursing care is the nurse's priority? A) Implementation of infection-control measures B) Close monitoring of skin integrity and color C) Frequent assessment of the patient's psychosocial status D) Administration of antiretroviral medications

Ans: A Feedback: Infection control is paramount following liver transplantation. This is a priority over skin integrity and psychosocial status, even though these are valid areas of assessment and intervention. Antiretrovirals are not indicated.

14. A patient is being discharged after a liver transplant and the nurse is performing discharge education. When planning this patient's continuing care, the nurse should prioritize which of the following risk diagnoses? A) Risk for Infection Related to Immunosuppressant Use B) Risk for Injury Related to Decreased Hemostasis C) Risk for Unstable Blood Glucose Related to Impaired Gluconeogenesis D) Risk for Contamination Related to Accumulation of Ammonia

Ans: A Feedback: Infection is the leading cause of death after liver transplantation. Pulmonary and fungal infections are common; susceptibility to infection is increased by the immunosuppressive therapy that is needed to prevent rejection. This risk exceeds the threats of injury and unstable blood glucose. The diagnosis of Risk for Contamination relates to environmental toxin exposure.

17. A patient has developed hepatic encephalopathy secondary to cirrhosis and is receiving care on the medical unit. The patient's current medication regimen includes lactulose (Cephulac) four times daily. What desired outcome should the nurse relate to this pharmacologic intervention? A) Two to 3 soft bowel movements daily B) Significant increase in appetite and food intake C) Absence of nausea and vomiting D) Absence of blood or mucus in stool

Ans: A Feedback: Lactulose (Cephulac) is administered to reduce serum ammonia levels. Two or three soft stools per day are desirable; this indicates that lactulose is performing as intended. Lactulose does not address the patient's appetite, symptoms of nausea and vomiting, or the development of blood and mucus in the stool.

37. A patient with ongoing back pain, nausea, and abdominal bloating has been diagnosed with cholecystitis secondary to gallstones. The nurse should anticipate that the patient will undergo what intervention? A) Laparoscopic cholecystectomy B) Methyl tertiary butyl ether (MTBE) infusion C) Intracorporeal lithotripsy D) Extracorporeal shock wave therapy (ESWL)

Ans: A Feedback: Most of the nonsurgical approaches, including lithotripsy and dissolution of gallstones, provide only temporary solutions to gallstone problems and are infrequently used in the United States. Cholecystectomy is the preferred treatment.

36. A patient with a cholelithiasis has been scheduled for a laparoscopic cholecystectomy. Why is laparoscopic cholecystectomy preferred by surgeons over an open procedure? A) Laparoscopic cholecystectomy poses fewer surgical risks than an open procedure. B) Laparoscopic cholecystectomy can be performed in a clinic setting, while an open procedure requires an OR. C) A laparoscopic approach allows for the removal of the entire gallbladder. D) A laparoscopic approach can be performed under conscious sedation.

Ans: A Feedback: Open surgery has largely been replaced by laparoscopic cholecystectomy (removal of the gallbladder through a small incision through the umbilicus). As a result, surgical risks have decreased, along with the length of hospital stay and the long recovery period required after standard surgical cholecystectomy. Both approaches allow for removal of the entire gallbladder and must be performed under general anesthetic in an operating theater.

6. A 37-year-old male patient presents at the emergency department (ED) complaining of nausea and vomiting and severe abdominal pain. The patient's abdomen is rigid, and there is bruising to the patient's flank. The patient's wife states that he was on a drinking binge for the past 2 days. The ED nurse should assist in assessing the patient for what health problem? A) Severe pancreatitis with possible peritonitis B) Acute cholecystitis C) Chronic pancreatitis D) Acute appendicitis with possible perforation

Ans: A Feedback: Severe abdominal pain is the major symptom of pancreatitis that causes the patient to seek medical care. Pain in pancreatitis is accompanied by nausea and vomiting that does not relieve the pain or nausea. Abdominal guarding is present and a rigid or board-like abdomen may be a sign of peritonitis. Ecchymosis (bruising) to the flank or around the umbilicus may indicate severe peritonitis. Pain generally occurs 24 to 48 hours after a heavy meal or alcohol ingestion. The link with alcohol intake makes pancreatitis a more likely possibility than appendicitis or cholecystitis.

7. A triage nurse in the emergency department is assessing a patient who presented with complaints of general malaise. Assessment reveals the presence of jaundice and increased abdominal girth. What assessment question best addresses the possible etiology of this patient's presentation? A) "How many alcoholic drinks do you typically consume in a week?" B) "To the best of your knowledge, are your immunizations up to date?" C) "Have you ever worked in an occupation where you might have been exposed to toxins?" D) "Has anyone in your family ever experienced symptoms similar to yours?"

Ans: A Feedback: Signs or symptoms of hepatic dysfunction indicate a need to assess for alcohol use. Immunization status, occupational risks, and family history are also relevant considerations, but alcohol use is a more common etiologic factor in liver disease.

19. A nurse is assisting with serving dinner trays on the unit. Upon receiving the dinner tray for a patient admitted with acute gallbladder inflammation, the nurse will question which of the following foods on the tray? A) Fried chicken B) Mashed potatoes C) Dinner roll D) Tapioca pudding

Ans: A Feedback: The diet immediately after an episode of acute cholecystitis is initially limited to low-fat liquids. Cooked fruits, rice or tapioca, lean meats, mashed potatoes, bread, and coffee or tea may be added as tolerated. The patient should avoid fried foods such as fried chicken, as fatty foods may bring on an episode of cholecystitis.

9. A nurse is caring for a patient with hepatic encephalopathy. While making the initial shift assessment, the nurse notes that the patient has a flapping tremor of the hands. The nurse should document the presence of what sign of liver disease? A) Asterixis B) Constructional apraxia C) Fetor hepaticus D) Palmar erythema

Ans: A Feedback: The nurse will document that a patient exhibiting a flapping tremor of the hands is demonstrating asterixis. While constructional apraxia is a motor disturbance, it is the inability to reproduce a simple figure. Fetor hepaticus is a sweet, slightly fecal odor to the breath and not associated with a motor disturbance. Skin changes associated with liver dysfunction may include palmar erythema, which is a reddening of the palms, but is not a flapping tremor.

39. A 55-year-old female patient with hepatocellular carcinoma (HCC) is undergoing radiofrequency ablation. The nurse should recognize what goal of this treatment? A) Destruction of the patient's liver tumor B) Restoration of portal vein patency C) Destruction of a liver abscess D) Reversal of metastasis

Ans: A Feedback: Using radiofrequency ablation, a tumor up to 5 cm in size can be destroyed in one treatment session. This technique does not address circulatory function or abscess formation. It does not allow for the reversal of metastasis.

4. A nurse educator is teaching a group of recent nursing graduates about their occupational risks for contracting hepatitis B. What preventative measures should the educator promote? Select all that apply. A) Immunization B) Use of standard precautions C) Consumption of a vitamin-rich diet D) Annual vitamin K injections E) Annual vitamin B12 injections

Ans: A, B Feedback: People who are at high risk, including nurses and other health care personnel exposed to blood or blood products, should receive active immunization. The consistent use of standard precautions is also highly beneficial. Vitamin supplementation is unrelated to an individual's risk of HBV.

23. A patient with pancreatic cancer has been scheduled for a pancreaticoduodenectomy (Whipple procedure). During health education, the patient should be informed that this procedure will involve the removal of which of the following? Select all that apply. A) Gallbladder B) Part of the stomach C) Duodenum D) Part of the common bile duct E) Part of the rectum

Ans: A, B, C, D Feedback: A pancreaticoduodenectomy (Whipple procedure or resection) is used for potentially resectable cancer of the head of the pancreas (Fig. 50-7). This procedure involves removal of the gallbladder, a portion of the stomach, duodenum, proximal jejunum, head of the pancreas, and distal common bile duct. The rectum is not affected.

3. A patient's assessment and diagnostic testing are suggestive of acute pancreatitis. When the nurse is performing the health interview, what assessment questions address likely etiologic factors? Select all that apply. A) "How many alcoholic drinks do you typically consume in a week?" B) "Have you ever been tested for diabetes?" C) "Have you ever been diagnosed with gallstones?" D) "Would you say that you eat a particularly high-fat diet?" E) "Does anyone in your family have cystic fibrosis?"

Ans: A, C Feedback: Eighty percent of patients with acute pancreatitis have biliary tract disease such as gallstones or a history of long-term alcohol abuse. Diabetes, high-fat consumption, and cystic fibrosis are not noted etiologic factors.

21. A patient's physician has ordered a "liver panel" in response to the patient's development of jaundice. When reviewing the results of this laboratory testing, the nurse should expect to review what blood tests? Select all that apply. A) Alanine aminotransferase (ALT) B) C-reactive protein (CRP) C) Gamma-glutamyl transferase (GGT) D) Aspartate aminotransferase (AST) E) B-type natriuretic peptide (BNP)

Ans: A, C, D Feedback: Liver function testing includes GGT, ALT, and AST. CRP addresses the presence of generalized inflammation and BNP is relevant to heart failure; neither is included in a liver panel.

33. A patient with end-stage liver disease has developed hypervolemia. What nursing interventions would be most appropriate when addressing the patient's fluid volume excess? Select all that apply. A) Administering diuretics B) Administering calcium channel blockers C) Implementing fluid restrictions D) Implementing a 1500 kcal/day restriction E) Enhancing patient positioning

Ans: A, C, E Feedback: Administering diuretics, implementing fluid restrictions, and enhancing patient positioning can optimize the management of fluid volume excess. Calcium channel blockers and calorie restriction do not address this problem.

26. A patient with a diagnosis of esophageal varices has undergone endoscopy to gauge the progression of this complication of liver disease. Following the completion of this diagnostic test, what nursing intervention should the nurse perform? A) Keep patient NPO until the results of test are known. B) Keep patient NPO until the patient's gag reflex returns. C) Administer analgesia until post-procedure tenderness is relieved. D) Give the patient a cold beverage to promote swallowing ability.

Ans: B Feedback: After the examination, fluids are not given until the patient's gag reflex returns. Lozenges and gargles may be used to relieve throat discomfort if the patient's physical condition and mental status permit. The result of the test is known immediately. Food and fluids are contraindicated until the gag reflex returns.

11. A participant in a health fair has asked the nurse about the role of drugs in liver disease. What health promotion teaching has the most potential to prevent drug-induced hepatitis? A) Finish all prescribed courses of antibiotics, regardless of symptom resolution. B) Adhere to dosing recommendations of OTC analgesics. C) Ensure that expired medications are disposed of safely. D) Ensure that pharmacists regularly review drug regimens for potential interactions.

Ans: B Feedback: Although any medication can affect liver function, use of acetaminophen (found in many over-the-counter medications used to treat fever and pain) has been identified as the leading cause of acute liver failure. Finishing prescribed antibiotics and avoiding expired medications are unrelated to this disease. Drug interactions are rarely the cause of drug-induced hepatitis.

24. A nurse is caring for a patient who has been admitted for the treatment of advanced cirrhosis. What assessment should the nurse prioritize in this patient's plan of care? A) Measurement of abdominal girth and body weight B) Assessment for variceal bleeding C) Assessment for signs and symptoms of jaundice D) Monitoring of results of liver function testing

Ans: B Feedback: Esophageal varices are a major cause of mortality in patients with uncompensated cirrhosis. Consequently, this should be a focus of the nurse's assessments and should be prioritized over the other listed assessments, even though each should be performed.

22. The nurse is caring for a patient who has just returned from the ERCP removal of gallstones. The nurse should monitor the patient for signs of what complications? A) Pain and peritonitis B) Bleeding and perforation C) Acidosis and hypoglycemia D) Gangrene of the gallbladder and hyperglycemia

Ans: B Feedback: Following ERCP removal of gallstones, the patient is observed closely for bleeding, perforation, and the development of pancreatitis or sepsis. Blood sugar alterations, gangrene, peritonitis, and acidosis are less likely complications.

14. A patient has had a laparoscopic cholecystectomy. The patient is now complaining of right shoulder pain. What should the nurse suggest to relieve the pain? A) Aspirin every 4 to 6 hours as ordered B) Application of heat 15 to 20 minutes each hour C) Application of an ice pack for no more than 15 minutes D) Application of liniment rub to affected area

Ans: B Feedback: If pain occurs in the right shoulder or scapular area (from migration of the CO2 used to insufflate the abdominal cavity during the procedure), the nurse may recommend use of a heating pad for 15 to 20 minutes hourly, walking, and sitting up when in bed. Aspirin would constitute a risk for bleeding.

19. A nurse is caring for a patient with a blocked bile duct from a tumor. What manifestation of obstructive jaundice should the nurse anticipate? A) Watery, blood-streaked diarrhea B) Orange and foamy urine C) Increased abdominal girth D) Decreased cognition

Ans: B Feedback: If the bile duct is obstructed, the bile will be reabsorbed into the blood and carried throughout the entire body. It is excreted in the urine, which becomes deep orange and foamy. Bloody diarrhea, ascites, and cognitive changes are not associated with obstructive jaundice.

10. A nurse is caring for a patient who has been scheduled for endoscopic retrograde cholangiopancreatography (ERCP) the following day. When providing anticipatory guidance for this patient, the nurse should describe what aspect of this diagnostic procedure? A) The need to protect the incision postprocedure B) The use of moderate sedation C) The need to infuse 50% dextrose during the procedure D) The use of general anesthesia

Ans: B Feedback: Moderate sedation, not general anesthesia, is used during ERCP. D50 is not administered and the procedure does not involve the creation of an incision.

32. A nurse is caring for a patient with cirrhosis secondary to heavy alcohol use. The nurse's most recent assessment reveals subtle changes in the patient's cognition and behavior. What is the nurse's most appropriate response? A) Ensure that the patient's sodium intake does not exceed recommended levels. B) Report this finding to the primary care provider due to the possibility of hepatic encephalopathy. C) Inform the primary care provider that the patient should be assessed for alcoholic hepatitis. D) Implement interventions aimed at ensuring a calm and therapeutic care environment.

Ans: B Feedback: Monitoring is an essential nursing function to identify early deterioration in mental status. The nurse monitors the patient's mental status closely and reports changes so that treatment of encephalopathy can be initiated promptly. This change in status is likely unrelated to sodium intake and would not signal the onset of hepatitis. A supportive care environment is beneficial, but does not address the patient's physiologic deterioration.

9. A patient with chronic pancreatitis had a pancreaticojejunostomy created 3 months ago for relief of pain and to restore drainage of pancreatic secretions. The patient has come to the office for a routine postsurgical appointment. The patient is frustrated that the pain has not decreased. What is the most appropriate initial response by the nurse? A) "The majority of patients who have a pancreaticojejunostomy have their normal digestion restored but do not achieve pain relief." B) "Pain relief occurs by 6 months in most patients who undergo this procedure, but some people experience a recurrence of their pain." C) "Your physician will likely want to discuss the removal of your gallbladder to achieve pain relief." D) "You are probably not appropriately taking the medications for your pancreatitis and pain, so we will need to discuss your medication regimen in detail."

Ans: B Feedback: Pain relief from a pancreaticojejunostomy often occurs by 6 months in more than 85% of the patients who undergo this procedure, but pain returns in a substantial number of patients as the disease progresses. This patient had surgery 3 months ago; the patient has 3 months before optimal benefits of the procedure may be experienced. There is no obvious indication for gallbladder removal and nonadherence is not the most likely factor underlying the pain.

30. A patient has been admitted to the critical care unit with a diagnosis of toxic hepatitis. When planning the patient's care, the nurse should be aware of what potential clinical course of this health problem? Place the following events in the correct sequence. 1. Fever rises. 2. Hematemesis. 3. Clotting abnormalities. 4. Vascular collapse. 5. Coma. A) 1, 2, 5, 4, 3 B) 1, 2, 3, 4, 5 C) 2, 3, 1, 4, 5 D) 3, 1, 2, 5, 4

Ans: B Feedback: Recovery from acute toxic hepatitis is rapid if the hepatotoxin is identified early and removed or if exposure to the agent has been limited. Recovery is unlikely if there is a prolonged period between exposure and onset of symptoms. There are no effective antidotes. The fever rises; the patient becomes toxic and prostrated. Vomiting may be persistent, with the emesis containing blood. Clotting abnormalities may be severe, and hemorrhages may appear under the skin. The severe GI symptoms may lead to vascular collapse. Delirium, coma, and seizures develop, and within a few days the patient may die of fulminant hepatic failure unless he or she receives a liver transplant.

13. A patient has been diagnosed with advanced stage breast cancer and will soon begin aggressive treatment. What assessment findings would most strongly suggest that the patient may have developed liver metastases? A) Persistent fever and cognitive changes B) Abdominal pain and hepatomegaly C) Peripheral edema unresponsive to diuresis D) Spontaneous bleeding and jaundice

Ans: B Feedback: The early manifestations of malignancy of the liver include pain—a continuous dull ache in the right upper quadrant, epigastrium, or back. Weight loss, loss of strength, anorexia, and anemia may also occur. The liver may be enlarged and irregular on palpation. Jaundice is present only if the larger bile ducts are occluded by the pressure of malignant nodules in the hilum of the liver. Fever, cognitive changes, peripheral edema, and bleeding are atypical signs.

31. A patient has been diagnosed with acute pancreatitis. The nurse is addressing the diagnosis of Acute Pain Related to Pancreatitis. What pharmacologic intervention is most likely to be ordered for this patient? A) Oral oxycodone B) IV hydromorphone (Dilaudid) C) IM meperidine (Demerol) D) Oral naproxen (Aleve)

Ans: B Feedback: The pain of acute pancreatitis is often very severe and pain relief may require parenteral opioids such as morphine, fentanyl (Sublimaze), or hydromorphone (Dilaudid). There is no clinical evidence to support the use of meperidine for pain relief in pancreatitis. Opioids are preferred over NSAIDs.

35. A patient with cirrhosis has experienced a progressive decline in his health; and liver transplantation is being considered by the interdisciplinary team. How will the patient's prioritization for receiving a donor liver be determined? A) By considering the patient's age and prognosis B) By objectively determining the patient's medical need C) By objectively assessing the patient's willingness to adhere to post-transplantation care D) By systematically ruling out alternative treatment options

Ans: B Feedback: The patient would undergo a classification of the degree of medical need through an objective determination known as the Model of End-Stage Liver Disease (MELD) classification, which stratifies the level of illness of those awaiting a liver transplant. This algorithm considers multiple variables, not solely age, prognosis, potential for adherence, and the rejection of alternative options.

28. A patient with a history of injection drug use has been diagnosed with hepatitis C. When collaborating with the care team to plan this patient's treatment, the nurse should anticipate what intervention? A) Administration of immune globulins B) A regimen of antiviral medications C) Rest and watchful waiting D) Administration of fresh-frozen plasma (FFP)

Ans: B Feedback: There is no benefit from rest, diet, or vitamin supplements in HCV treatment. Studies have demonstrated that a combination of two antiviral agents, Peg-interferon and ribavirin (Rebetol), is effective in producing improvement in patients with hepatitis C and in treating relapses. Immune globulins and FFP are not indicated.

38. A nurse is caring for a patient with gallstones who has been prescribed ursodeoxycholic acid (UDCA). The patient askshow this medicine is going to help his symptoms. The nurse should be aware of what aspect of this drug's pharmacodynamics? A) It inhibits the synthesis of bile. B) It inhibits the synthesis and secretion of cholesterol. C) It inhibits the secretion of bile. D) It inhibits the synthesis and secretion of amylase.

Ans: B Feedback: UDCA acts by inhibiting the synthesis and secretion of cholesterol, thereby desaturating bile. UDCA does not directly inhibit either the synthesis or secretion of bile or amylase.

35. A patient is admitted to the unit with acute cholecystitis. The physician has noted that surgery will be scheduled in 4 days. The patient asks why the surgery is being put off for a week when he has a "sick gallbladder." What rationale would underlie the nurse's response? A) Surgery is delayed until the patient can eat a regular diet without vomiting. B) Surgery is delayed until the acute symptoms subside. C) The patient requires aggressive nutritional support prior to surgery. D) Time is needed to determine whether a laparoscopic procedure can be used.

Ans: B Feedback: Unless the patient's condition deteriorates, surgical intervention is delayed just until the acute symptoms subside (usually within a few days). There is no need to delay surgery pending an improvement in nutritional status, and deciding on a laparoscopic approach is not a lengthy process.

18. A patient with gallstones has been prescribed ursodeoxycholic acid (UDCA). The nurse understands that additional teaching is needed regarding this medication when the patient states: A) "It is important that I see my physician for scheduled follow-up appointments while taking this medication." B) "I will take this medication for 2 weeks and then gradually stop taking it." C) "If I lose weight, the dose of the medication may need to be changed." D) "This medication will help dissolve small gallstones made of cholesterol."

Ans: B Feedback: Ursodeoxycholic acid (UDCA) has been used to dissolve small, radiolucent gallstones composed primarily of cholesterol. This drug can reduce the size of existing stones, dissolve small stones, and prevent new stones from forming. Six to 12 months of therapy is required in many patients to dissolve stones, and monitoring of the patient is required during this time. The effective dose of medication depends on body weight.

12. A nurse is preparing a plan of care for a patient with pancreatic cysts that have necessitated drainage through the abdominal wall. What nursing diagnosis should the nurse prioritize? A) Disturbed Body Image B) Impaired Skin Integrity C) Nausea D) Risk for Deficient Fluid Volume

Ans: B Feedback: While each of the diagnoses may be applicable to a patient with pancreatic drainage, the priority nursing diagnosis is Impaired Skin Integrity. The drainage is often perfuse and destructive to tissue because of the enzyme contents. Nursing measures must focus on steps to protect the skin near the drainage site from excoriation. The application of ointments or the use of a suction apparatus protects the skin from excoriation.

36. A nurse has entered the room of a patient with cirrhosis and found the patient on the floor. The patient states that she fell when transferring to the commode. The patient's vital signs are within reference ranges and the nurse observes no apparent injuries. What is the nurse's most appropriate action? A) Remove the patient's commode and supply a bedpan. B) Complete an incident report and submit it to the unit supervisor. C) Have the patient assessed by the physician due to the risk of internal bleeding. D) Perform a focused abdominal assessment in order to rule out injury.

Ans: C Feedback: A fall would necessitate thorough medical assessment due to the patient's risk of bleeding. The nurse's abdominal assessment is an appropriate action, but is not wholly sufficient to rule out internal injury. Medical assessment is a priority over removing the commode or filling out an incident report, even though these actions are appropriate.

11. A patient has undergone a laparoscopic cholecystectomy and is being prepared for discharge home. When providing health education, the nurse should prioritize which of the following topics? A) Management of fluid balance in the home setting B) The need for blood glucose monitoring for the next week C) Signs and symptoms of intra-abdominal complications D) Appropriate use of prescribed pancreatic enzymes

Ans: C Feedback: Because of the early discharge following laparoscopic cholecystectomy, the patient needs thorough education in the signs and symptoms of complications. Fluid balance is not typically a problem in the recovery period after laparoscopic cholecystectomy. There is no need for blood glucose monitoring or pancreatic enzymes.

32. A patient has just been diagnosed with chronic pancreatitis. The patient is underweight and in severe pain and diagnostic testing indicates that over 80% of the patient's pancreas has been destroyed. The patient asks the nurse why the diagnosis was not made earlier in the disease process. What would be the nurse's best response? A) "The symptoms of pancreatitis mimic those of much less serious illnesses." B) "Your body doesn't require pancreatic function until it is under great stress, so it is easy to go unnoticed." C) "Chronic pancreatitis often goes undetected until a large majority of pancreatic function is lost." D) "It's likely that your other organs were compensating for your decreased pancreatic function."

Ans: C Feedback: By the time symptoms occur in chronic pancreatitis, approximately 90% of normal acinar cell function (exocrine function) has been lost. Late detection is not usually attributable to the vagueness of symptoms. The pancreas contributes continually to homeostasis and other organs are unable to perform its physiologic functions.

25. A community health nurse is caring for a patient whose multiple health problems include chronic pancreatitis. During the most recent visit, the nurse notes that the patient is experiencing severe abdominal pain and has vomited 3 times in the past several hours. What is the nurse's most appropriate action? A) Administer a PRN dose of pancreatic enzymes as ordered. B) Teach the patient about the importance of abstaining from alcohol. C) Arrange for the patient to be transported to the hospital. D) Insert an NG tube, if available, and stay with the patient.

Ans: C Feedback: Chronic pancreatitis is characterized by recurring attacks of severe upper abdominal and back pain, accompanied by vomiting. The onset of these acute symptoms warrants hospital treatment. Pancreatic enzymes are not indicated and an NG tube would not be inserted in the home setting. Patient education is a later priority that may or may not be relevant.

34. A patient is admitted to the ICU with acute pancreatitis. The patient's family asks what causes acute pancreatitis. The critical care nurse knows that a majority of patients with acute pancreatitis have what? A) Type 1 diabetes B) An impaired immune system C) Undiagnosed chronic pancreatitis D) An amylase deficiency

Ans: C Feedback: Eighty percent of patients with acute pancreatitis have biliary tract disease or a history of long-term alcohol abuse. These patients usually have had undiagnosed chronic pancreatitis before their first episode of acute pancreatitis. Diabetes, an impaired immune function, and amylase deficiency are not specific precursors to acute pancreatitis.

5. A nurse who provides care in a walk-in clinic assesses a wide range of individuals. The nurse should identify which of the following patients as having the highest risk for chronic pancreatitis? A) A 45-year-old obese woman with a high-fat diet B) An 18-year-old man who is a weekend binge drinker C) A 39-year-old man with chronic alcoholism D) A 51-year-old woman who smokes one-and-a-half packs of cigarettes per day

Ans: C Feedback: Excessive and prolonged consumption of alcohol accounts for approximately 70% to 80% of all cases of chronic pancreatitis.

26. A student nurse is caring for a patient who has a diagnosis of acute pancreatitis and who is receiving parenteral nutrition. The student should prioritize which of the following assessments? A) Fluid output B) Oral intake C) Blood glucose levels D) BUN and creatinine levels

Ans: C Feedback: In addition to administering enteral or parenteral nutrition, the nurse monitors serum glucose levels every 4 to 6 hours. Output should be monitored but in most cases it is not more important than serum glucose levels. A patient on parenteral nutrition would have no oral intake to monitor. Blood sugar levels are more likely to be unstable than indicators of renal function.

29. The family of a patient in the ICU diagnosed with acute pancreatitis asks the nurse why the patient has been moved to an air bed. What would be the nurse's best response? A) "Air beds allow the care team to reposition her more easily while she's on bed rest." B) "Air beds are far more comfortable than regular beds and she'll likely have to be on bed rest a long time." C) "The bed automatically moves, so she's less likely to develop pressure sores while she's in bed." D) "The bed automatically moves, so she is likely to have less pain."

Ans: C Feedback: It is important to turn the patient every 2 hours; use of specialty beds may be indicated to prevent skin breakdown. The rationale for a specialty bed is not related to repositioning, comfort, or ease of movement.

31. A previously healthy adult's sudden and precipitous decline in health has been attributed to fulminant hepatic failure, and the patient has been admitted to the intensive care unit. The nurse should be aware that the treatment of choice for this patient is what? A) IV administration of immune globulins B) Transfusion of packed red blood cells and fresh-frozen plasma (FFP) C) Liver transplantation D) Lobectomy

Ans: C Feedback: Liver transplantation carries the highest potential for the resolution of fulminant hepatic failure. This is preferred over other interventions, such as pharmacologic treatments, transfusions, and surgery.

3. A patient with portal hypertension has been admitted to the medical floor. The nurse should prioritize which of the following assessments related to the manifestations of this health problem? A) Assessment of blood pressure and assessment for headaches and visual changes B) Assessments for signs and symptoms of venous thromboembolism C) Daily weights and abdominal girth measurement D) Blood glucose monitoring q4h

Ans: C Feedback: Obstruction to blood flow through the damaged liver results in increased blood pressure (portal hypertension) throughout the portal venous system. This can result in varices and ascites in the abdominal cavity. Assessments related to ascites are daily weights and abdominal girths. Portal hypertension is not synonymous with cardiovascular hypertension and does not create a risk for unstable blood glucose or VTE.

30. A patient is receiving care in the intensive care unit for acute pancreatitis. The nurse is aware that pancreatic necrosis is a major cause of morbidity and mortality in patients with acute pancreatitis. Consequently, the nurse should assess for what signs or symptoms of this complication? A) Sudden increase in random blood glucose readings B) Increased abdominal girth accompanied by decreased level of consciousness C) Fever, increased heart rate and decreased blood pressure D) Abdominal pain unresponsive to analgesics

Ans: C Feedback: Pancreatic necrosis is a major cause of morbidity and mortality in patients with acute pancreatitis because of resulting hemorrhage, septic shock, and multiple organ dysfunction syndrome (MODS). Signs of shock would include hypotension, tachycardia and fever. Each of the other listed changes in status warrants intervention, but none is clearly suggestive of an onset of pancreatic necrosis.

16. A nurse is caring for a patient with hepatic encephalopathy. The nurse's assessment reveals that the patient exhibits episodes of confusion, is difficult to arouse from sleep and has rigid extremities. Based on these clinical findings, the nurse should document what stage of hepatic encephalopathy? A) Stage 1 B) Stage 2 C) Stage 3 D) Stage 4

Ans: C Feedback: Patients in the third stage of hepatic encephalopathy exhibit the following symptoms: stuporous, difficult to arouse, sleeps most of the time, exhibits marked confusion, incoherent in speech, asterixis, increased deep tendon reflexes, rigidity of extremities, marked EEG abnormalities. Patients in stages 1 and 2 exhibit clinical symptoms that are not as advanced as found in stage 3, and patients in stage 4 are comatose. In stage 4, there is an absence of asterixis, absence of deep tendon reflexes, flaccidity of extremities, and EEG abnormalities.

22. A patient with liver disease has developed jaundice; the nurse is collaborating with the patient to develop a nutritional plan. The nurse should prioritize which of the following in the patient's plan? A) Increased potassium intake B) Fluid restriction to 2 L per day C) Reduction in sodium intake D) High-protein, low-fat diet

Ans: C Feedback: Patients with ascites require a sharp reduction in sodium intake. Potassium intake should not be correspondingly increased. There is no need for fluid restriction or increased protein intake.

40. A nurse is caring for a patient with severe hemolytic jaundice. Laboratory tests show free bilirubin to be 24 mg/dL. For what complication is this patient at risk? A) Chronic jaundice B) Pigment stones in portal circulation C) Central nervous system damage D) Hepatomegaly

Ans: C Feedback: Prolonged jaundice, even if mild, predisposes to the formation of pigment stones in the gallbladder, and extremely severe jaundice (levels of free bilirubin exceeding 20 to 25 mg/dL) poses a risk for CNS damage. There are not specific risks of hepatomegaly or chronic jaundice resulting from high bilirubin.

12. Diagnostic testing has revealed that a patient's hepatocellular carcinoma (HCC) is limited to one lobe. The nurse should anticipate that this patient's plan of care will focus on what intervention? A) Cryosurgery B) Liver transplantation C) Lobectomy D) Laser hyperthermia

Ans: C Feedback: Surgical resection is the treatment of choice when HCC is confined to one lobe of the liver and the function of the remaining liver is considered adequate for postoperative recovery. Removal of a lobe of the liver (lobectomy) is the most common surgical procedure for excising a liver tumor. While cryosurgery and liver transplantation are other surgical options for management of liver cancer, these procedures are not performed at the same frequency as a lobectomy. Laser hyperthermia is a nonsurgical treatment for liver cancer.

21. A nurse is creating a care plan for a patient with acute pancreatitis. The care plan includes reduced activity. What rationale for this intervention should be cited in the care plan? A) Bed rest reduces the patient's metabolism and reduces the risk of metabolic acidosis. B) Reduced activity protects the physical integrity of pancreatic cells. C) Bed rest lowers the metabolic rate and reduces enzyme production. D) Inactivity reduces caloric need and gastrointestinal motility.

Ans: C Feedback: The acutely ill patient is maintained on bed rest to decrease the metabolic rate and reduce the secretion of pancreatic and gastric enzymes. Staying in bed does not release energy from the body to fight the disease.

8. A patient who had surgery for gallbladder disease has just returned to the postsurgical unit from postanesthetic recovery. The nurse caring for this patient knows to immediately report what assessment finding to the physician? A) Decreased breath sounds B) Drainage of bile-colored fluid onto the abdominal dressing C) Rigidity of the abdomen D) Acute pain with movement

Ans: C Feedback: The location of the subcostal incision will likely cause the patient to take shallow breaths to prevent pain, which may result in decreased breath sounds. The nurse should remind patients to take deep breaths and cough to expand the lungs fully and prevent atelectasis. Acute pain is an expected assessment finding following surgery; analgesics should be administered for pain relief. Abdominal splinting or application of an abdominal binder may assist in reducing the pain. Bile may continue to drain from the drainage tract after surgery, which will require frequent changes of the abdominal dressing. Increased abdominal tenderness and rigidity should be reported immediately to the physician, as it may indicate bleeding from an inadvertent puncture or nicking of a major blood vessel during the surgical procedure.

39. A nurse is providing discharge education to a patient who has undergone a laparoscopic cholecystectomy. During the immediate recovery period, the nurse should recommend what foods? A) High-fiber foods B) Low-purine, nutrient-dense foods C) Low-fat foods high in proteins and carbohydrates D) Foods that are low-residue and low in fat

Ans: C Feedback: The nurse encourages the patient to eat a diet that is low in fats and high in carbohydrates and proteins immediately after surgery. There is no specific need to increase fiber or avoid purines. A low-residue diet is not indicated.

6. A patient who has undergone liver transplantation is ready to be discharged home. Which outcome of health education should the nurse prioritize? A) The patient will obtain measurement of drainage from the T-tube. B) The patient will exercise three times a week. C) The patient will take immunosuppressive agents as required. D) The patient will monitor for signs of liver dysfunction.

Ans: C Feedback: The patient is given written and verbal instructions about immunosuppressive agent doses and dosing schedules. The patient is also instructed on steps to follow to ensure that an adequate supply of medication is available so that there is no chance of running out of the medication or skipping a dose. Failure to take medications as instructed may precipitate rejection. The nurse would not teach the patient to measure drainage from a T-tube as the patient wouldn't go home with a T-tube. The nurse may teach the patient about the need to exercise or what the signs of liver dysfunction are, but the nurse would not stress these topics over the immunosuppressive drug regimen.

28. A patient has been admitted to the hospital for the treatment of chronic pancreatitis. The patient has been stabilized and the nurse is now planning health promotion and educational interventions. Which of the following should the nurse prioritize? A) Educating the patient about expectations and care following surgery B) Educating the patient about the management of blood glucose after discharge C) Educating the patient about postdischarge lifestyle modifications D) Educating the patient about the potential benefits of pancreatic transplantation

Ans: C Feedback: The patient's lifestyle (especially regarding alcohol use) is a major determinant of the course of chronic pancreatitis. The disease is not often managed by surgery and blood sugar monitoring is not necessarily indicated for every patient after hospital treatment. Transplantation is not an option.

25. A patient with a diagnosis of cirrhosis has developed variceal bleeding and will imminently undergo variceal banding. What psychosocial nursing diagnosis should the nurse most likely prioritize during this phase of the patient's treatment? A) Decisional Conflict B) Deficient Knowledge C) Death Anxiety D) Disturbed Thought Processes

Ans: C Feedback: The sudden hemorrhage that accompanies variceal bleeding is intensely anxiety-provoking. The nurse must address the patient's likely fear of death, which is a realistic possibility. For most patients, anxiety is likely to be a more acute concern than lack of knowledge or decisional conflict. The patient may or may not experience disturbances in thought processes.

2. A nurse is performing an admission assessment of a patient with a diagnosis of cirrhosis. What technique should the nurse use to palpate the patient's liver? A) Place hand under the right lower abdominal quadrant and press down lightly with the other hand. B) Place the left hand over the abdomen and behind the left side at the 11th rib. C) Place hand under right lower rib cage and press down lightly with the other hand. D) Hold hand 90 degrees to right side of the abdomen and push down firmly.

Ans: C Feedback: To palpate the liver, the examiner places one hand under the right lower rib cage and presses downward with light pressure with the other hand. The liver is not on the left side or in the right lower abdominal quadrant.

16. A patient has been treated in the hospital for an episode of acute pancreatitis. The patient has acknowledged the role that his alcohol use played in the development of his health problem, but has not expressed specific plans for lifestyle changes after discharge. What is the nurse's most appropriate response? A) Educate the patient about the link between alcohol use and pancreatitis. B) Ensure that the patient knows the importance of attending follow-up appointments. C) Refer the patient to social work or spiritual care. D) Encourage the patient to connect with a community-based support group.

Ans: D Feedback: After the acute attack has subsided, some patients may be inclined to return to their previous drinking habits. The nurse provides specific information about resources and support groups that may be of assistance in avoiding alcohol in the future. Referral to Alcoholics Anonymous as appropriate or other support groups is essential. The patient already has an understanding of the effects of alcohol, and follow-up appointments will not necessarily result in lifestyle changes. Social work and spiritual care may or may not be beneficial.

2. A 55-year-old man has been newly diagnosed with acute pancreatitis and admitted to the acute medical unit. How should the nurse most likely explain the pathophysiology of this patient's health problem? A) "Toxins have accumulated and inflamed your pancreas." B) "Bacteria likely migrated from your intestines and became lodged in your pancreas." C) "A virus that was likely already present in your body has begun to attack your pancreatic cells." D) "The enzymes that your pancreas produces have damaged the pancreas itself."

Ans: D Feedback: Although the mechanisms causing pancreatitis are unknown, pancreatitis is commonly described as the autodigestion of the pancreas. Less commonly, toxic substances and microorganisms are implicated as the cause of pancreatitis.

27. A patient with esophageal varices is being cared for in the ICU. The varices have begun to bleed and the patient is at risk for hypovolemia. The patient has Ringer's lactate at 150 cc/hr infusing. What else might the nurse expect to have ordered to maintain volume for this patient? A) Arterial line B) Diuretics C) Foley catheter D) Volume expanders

Ans: D Feedback: Because patients with bleeding esophageal varices have intravascular volume depletion and are subject to electrolyte imbalance, IV fluids with electrolytes and volume expanders are provided to restore fluid volume and replace electrolytes. Diuretics would reduce vascular volume. An arterial line and Foley catheter are likely to be ordered, but neither actively maintains the patient's volume.

1. A nurse is caring for a patient with liver failure and is performing an assessment in the knowledge of the patient's increased risk of bleeding. The nurse recognizes that this risk is related to the patient's inability to synthesize prothrombin in the liver. What factor most likely contributes to this loss of function? A) Alterations in glucose metabolism B) Retention of bile salts C) Inadequate production of albumin by hepatocytes D) Inability of the liver to use vitamin K

Ans: D Feedback: Decreased production of several clotting factors may be partially due to deficient absorption of vitamin K from the GI tract. This probably is caused by the inability of liver cells to use vitamin K to make prothrombin. This bleeding risk is unrelated to the roles of glucose, bile salts, or albumin.

34. A patient with liver cancer is being discharged home with a biliary drainage system in place. The nurse should teach the patient's family how to safely perform which of the following actions? A) Aspirating bile from the catheter using a syringe B) Removing the catheter when output is 15 mL in 24 hours C) Instilling antibiotics into the catheter D) Assessing the patency of the drainage catheter

Ans: D Feedback: Families should be taught to provide basic catheter care, including assessment of patency. Antibiotics are not instilled into the catheter and aspiration using a syringe is contraindicated. The family would not independently remove the catheter; this would be done by a member of the care team when deemed necessary.

10. A local public health nurse is informed that a cook in a local restaurant has been diagnosed with hepatitis A. What should the nurse advise individuals to obtain who ate at this restaurant and have never received the hepatitis A vaccine? A) The hepatitis A vaccine B) Albumin infusion C) The hepatitis A and B vaccines D) An immune globulin injection

Ans: D Feedback: For people who have not been previously vaccinated, hepatitis A can be prevented by the intramuscular administration of immune globulin during the incubation period, if given within 2 weeks of exposure. Administration of the hepatitis A vaccine will not protect the patient exposed to hepatitis A, as protection will take a few weeks to develop after the first dose of the vaccine. The hepatitis B vaccine provides protection again the hepatitis B virus, but plays no role in protection for the patient exposed to hepatitis A. Albumin confers no therapeutic benefit.

40. A patient presents to the emergency department (ED) complaining of severe right upper quadrant pain. The patient states that his family doctor told him he had gallstones. The ED nurse should recognize what possible complication of gallstones? A) Acute pancreatitis B) Atrophy of the gallbladder C) Gallbladder cancer D) Gangrene of the gallbladder

Ans: D Feedback: In calculous cholecystitis, a gallbladder stone obstructs bile outflow. Bile remaining in the gallbladder initiates a chemical reaction; autolysis and edema occur; and the blood vessels in the gallbladder are compressed, compromising its vascular supply. Gangrene of the gallbladder with perforation may result. Pancreatitis, atrophy, and cancer of the gallbladder are not plausible complications.

37. A patient with liver cancer is being discharged home with a hepatic artery catheter in place. The nurse should be aware that this catheter will facilitate which of the following? A) Continuous monitoring for portal hypertension B) Administration of immunosuppressive drugs during the first weeks after transplantation C) Real-time monitoring of vascular changes in the hepatic system D) Delivery of a continuous chemotherapeutic dose

Ans: D Feedback: In most cases, the hepatic artery catheter has been inserted surgically and has a prefilled infusion pump that delivers a continuous chemotherapeutic dose until completed. The hepatic artery catheter does not monitor portal hypertension, deliver immunosuppressive drugs, or monitor vascular changes in the hepatic system.

8. A nurse is participating in the emergency care of a patient who has just developed variceal bleeding. What intervention should the nurse anticipate? A) Infusion of intravenous heparin B) IV administration of albumin C) STAT administration of vitamin K by the intramuscular route D) IV administration of octreotide (Sandostatin)

Ans: D Feedback: Octreotide (Sandostatin)—a synthetic analog of the hormone somatostatin—is effective in decreasing bleeding from esophageal varices, and lacks the vasoconstrictive effects of vasopressin. Because of this safety and efficacy profile, octreotide is considered the preferred treatment regimen for immediate control of variceal bleeding. Vitamin K and albumin are not administered and heparin would exacerbate, not alleviate, bleeding.

33. A patient has been diagnosed with pancreatic cancer and has been admitted for care. Following initial treatment, the nurse should be aware that the patient is most likely to require which of the following? A) Inpatient rehabilitation B) Rehabilitation in the home setting C) Intensive physical therapy D) Hospice care

Ans: D Feedback: Pancreatic carcinoma has only a 5% survival rate at 5 years regardless of the stage of disease at diagnosis or treatment. As a result, there is a higher likelihood that the patient will require hospice care than physical therapy and rehabilitation.

20. A nurse is assessing an elderly patient with gallstones. The nurse is aware that the patient may not exhibit typical symptoms, and that particular symptoms that may be exhibited in the elderly patient may include what? A) Fever and pain B) Chills and jaundice C) Nausea and vomiting D) Signs and symptoms of septic shock

Ans: D Feedback: The elderly patient may not exhibit the typical symptoms of fever, pain, chills jaundice, and nausea and vomiting. Symptoms of biliary tract disease in the elderly may be accompanied or preceded by those of septic shock, which include oliguria, hypotension, change in mental status, tachycardia, and tachypnea.

18. A nurse is performing an admission assessment for an 81-year-old patient who generally enjoys good health. When considering normal, age-related changes to hepatic function, the nurse should anticipate what finding? A) Similar liver size and texture as in younger adults B) A nonpalpable liver C) A slightly enlarged liver with palpably hard edges D) A slightly decreased size of the liver

Ans: D Feedback: The most common age-related change in the liver is a decrease in size and weight. The liver is usually still palpable, however, and is not expected to have hardened edges.

15. A patient returns to the floor after a laparoscopic cholecystectomy. The nurse should assess the patient for signs and symptoms of what serious potential complication of this surgery? A) Diabetic coma B) Decubitus ulcer C) Wound evisceration D) Bile duct injury

Ans: D Feedback: The most serious complication after laparoscopic cholecystectomy is a bile duct injury. Patients do not face a risk of diabetic coma. A decubitus ulcer is unlikely because immobility is not expected. Evisceration is highly unlikely, due to the laparoscopic approach.

13. A home health nurse is caring for a patient discharged home after pancreatic surgery. The nurse documents the nursing diagnosis Risk for Imbalanced Nutrition: Less than Body Requirements on the care plan based on the potential complications that may occur after surgery. What are the most likely complications for the patient who has had pancreatic surgery? A) Proteinuria and hyperkalemia B) Hemorrhage and hypercalcemia C) Weight loss and hypoglycemia D) Malabsorption and hyperglycemia

Ans: D Feedback: The nurse arrives at this diagnosis based on the complications of malabsorption and hyperglycemia. These complications often lead to the need for dietary modifications. Pancreatic enzyme replacement, a low-fat diet, and vitamin supplementation often are also required to meet the patient's nutritional needs and restrictions. Electrolyte imbalances often accompany pancreatic disorders and surgery, but the electrolyte levels are more often deficient than excessive. Hemorrhage is a complication related to surgery, but not specific to the nutritionally based nursing diagnosis. Weight loss is a common complication, but hypoglycemia is less likely.

17. A patient is being treated on the acute medical unit for acute pancreatitis. The nurse has identified a diagnosis of Ineffective Breathing Pattern Related to Pain. What intervention should the nurse perform in order to best address this diagnosis? A) Position the patient supine to facilitate diaphragm movement. B) Administer corticosteroids by nebulizer as ordered. C) Perform oral suctioning as needed to remove secretions. D) Maintain the patient in a semi-Fowler's position whenever possible.

Ans: D Feedback: The nurse maintains the patient in a semi-Fowler's position to decrease pressure on the diaphragm by a distended abdomen and to increase respiratory expansion. A supine position will result in increased pressure on the diaphragm and potentially decreased respiratory expansion. Steroids and oral suctioning are not indicated.

1. A nurse is assessing a patient who has been diagnosed with cholecystitis, and is experiencing localized abdominal pain. When assessing the characteristics of the patient's pain, the nurse should anticipate that it may radiate to what region? A) Left upper chest B) Inguinal region C) Neck or jaw D) Right shoulder

Ans: D Feedback: The patient may have biliary colic with excruciating upper right abdominal pain that radiates to the back or right shoulder. Pain from cholecystitis does not typically radiate to the left upper chest, inguinal area, neck, or jaw.

23. A nurse is amending a patient's plan of care in light of the fact that the patient has recently developed ascites. What should the nurse include in this patient's care plan? A) Mobilization with assistance at least 4 times daily B) Administration of beta-adrenergic blockers as ordered C) Vitamin B12 injections as ordered D) Administration of diuretics as ordered

Ans: D Feedback: Use of diuretics along with sodium restriction is successful in 90% of patients with ascites. Beta-blockers are not used to treat ascites and bed rest is often more beneficial than increased mobility. Vitamin B12 injections are not necessary.

A nurse is working with a patient who has chronic constipation. What should be included in patient teaching to promote normal bowel function? A)Use glycerin suppositories on a regular basis. B)Limit physical activity in order to promote bowel peristalsis. C)Consume high-residue, high-fiber foods. D)Resist the urge to defecate until the urge becomes intense.

Ans:C Feedback:Goals for the patient include restoring or maintaining a regular pattern of elimination by responding to the urge to defecate, ensuring adequate intake of fluids and high-fiber foods, learning about methods to avoid constipation, relieving anxiety about bowel elimination patterns, and avoiding complications. Ongoing use of pharmacologic aids should not be promoted, due to the risk of dependence. Increased mobility helps to maintain a regular pattern of elimination. The urge to defecate should be heeded.

When caring for a client with an acute exacerbation of a peptic ulcer, the nurse finds the client doubled up in bed with severe pain to his right shoulder. The intial appropriate action by the nurse is to

Assess the client's abdomen and vital signs.

The nurse is caring for a patient with cirrhosis of the liver and observes that the patient is having hand-flapping tremors. What does the nurse document this finding as? a) Fetor hepaticus b) Constructional apraxia c) Ataxia d) Asterixis

Asterixis Asterixis, an involuntary flapping of the hands, may be seen in stage II encephalopathy (Fig. 49-13).

teaching for client with new diagnosis of acute pylenephritis

Avoid NSAIDS for pain

When talking with a patient about taking loperamide (Imodium), the health care professional should include which of the following instructions? Dissolve the powder thoroughly in 8 oz of water. Have diphenhydramine available. Avoid activities that require alertness. Take 30 min before activities that trigger nausea.

Avoid activities that require alertness. rationale: Loperamide, an opioid agonist, can cause sedation and dizziness. The health care professional should caution the patient about not taking it before activities that require alertness.

1. A nurse is caring for a patient who just has been diagnosed with a peptic ulcer. When teaching the patient about his new diagnosis, how should the nurse best describe a peptic ulcer? A) Inflammation of the lining of the stomach B) Erosion of the lining of the stomach or intestine C) Bleeding from the mucosa in the stomach D) Viral invasion of the stomach wall

B Feedback: A peptic ulcer is erosion of the lining of the stomach or intestine. Peptic ulcers are often accompanied by bleeding and inflammation, but these are not the definitive characteristics.

27. A patient with a history of peptic ulcer disease has presented to the emergency department (ED) in distress. What assessment finding would lead the ED nurse to suspect that the patient has a perforated ulcer? A) The patient has abdominal bloating that developed rapidly. B) The patient has a rigid, "boardlike" abdomen that is tender. C) The patient is experiencing intense lower right quadrant pain. D) The patient is experiencing dizziness and confusion with no apparent hemodynamic changes.

B Feedback: An extremely tender and rigid (boardlike) abdomen is suggestive of a perforated ulcer. None of the other listed signs and symptoms is suggestive of a perforated ulcer.

6. A nurse caring for a patient who has had bariatric surgery is developing a teaching plan in anticipation of the patient's discharge. Which of the following is essential to include? A) Drink a minimum of 12 ounces of fluid with each meal. B) Eat several small meals daily spaced at equal intervals. C) Choose foods that are high in simple carbohydrates. D) Sit upright when eating and for 30 minutes afterward.

B Feedback: Due to decreased stomach capacity, the patient must consume small meals at intervals to meet nutritional requirements while avoiding a feeling of fullness and complications such as dumping syndrome. The patient should not consume fluids with meals and low-Fowler's positioning is recommended during and after meals. Carbohydrates should be limited.

34. A patient has recently received a diagnosis of gastric cancer; the nurse is aware of the importance of assessing the patient's level of anxiety. Which of the following actions is most likely to accomplish this? A) The nurse gauges the patient's response to hypothetical outcomes. B) The patient is encouraged to express fears openly. C) The nurse provides detailed and accurate information about the disease. D) The nurse closely observes the patient's body language.

B Feedback: Encouraging the patient to discuss his or her fears and anxieties is usually the best way to assess a patient's anxiety. Presenting hypothetical situations is a surreptitious and possibly inaccurate way of assessing anxiety. Observing body language is part of assessment, but it is not the complete assessment. Presenting information may alleviate anxiety for some patients, but it is not an assessment.

26. A patient has been admitted to the hospital after diagnostic imaging revealed the presence of a gastric outlet obstruction (GOO). What is the nurse's priority intervention? A) Administration of antiemetics B) Insertion of an NG tube for decompression C) Infusion of hypotonic IV solution D) Administration of proton pump inhibitors as ordered

B Feedback: In treating the patient with gastric outlet obstruction, the first consideration is to insert an NG tube to decompress the stomach. This is a priority over fluid or medication administration.

33. A patient who is obese is exploring bariatric surgery options and presented to a bariatric clinic for preliminary investigation. The nurse interviews the patient, analyzing and documenting the data. Which of the following nursing diagnoses may be a contraindication for bariatric surgery? A) Disturbed Body Image Related to Obesity B) Deficient Knowledge Related to Risks and Expectations of Surgery C) Anxiety Related to Surgery D) Chronic Low Self-Esteem Related to Obesity

B Feedback: It is expected that patients seeking bariatric surgery may have challenges with body image and self-esteem related to their obesity. Anxiety is also expected when facing surgery. However, if the patient's knowledge remains deficient regarding the risks and realistic expectations for surgery, this may show that the patient is not an appropriate surgical candidate.

10. A nurse is assessing a patient who has peptic ulcer disease. The patient requests more information about the typical causes of Helicobacter pylori infection. What would it be appropriate for the nurse to instruct the patient? A) Most affected patients acquired the infection during international travel. B) Infection typically occurs due to ingestion of contaminated food and water. C) Many people possess genetic factors causing a predisposition to H. pylori infection. D) The H. pylori microorganism is endemic in warm, moist climates.

B Feedback: Most peptic ulcers result from infection with the gram-negative bacteria H. pylori, which may be acquired through ingestion of food and water. The organism is endemic to all areas of the United States. Genetic factors have not been identified.

19. A patient comes to the bariatric clinic to obtain information about bariatric surgery. The nurse assesses the obese patient knowing that in addition to meeting the criterion of morbid obesity, a candidate for bariatric surgery must also demonstrate what? A) Knowledge of the causes of obesity and its associated risks B) Adequate understanding of required lifestyle changes C) Positive body image and high self-esteem D) Insight into why past weight loss efforts failed

B Feedback: Patients seeking bariatric surgery should be free of serious mental disorders and motivated to comply with lifestyle changes related to eating patterns, dietary choices, and elimination. While assessment of knowledge about causes of obesity and its associated risks as well as insight into the reasons why previous diets have been ineffective are included in the client's plan of care, these do not predict positive client outcomes following bariatric surgery. Most obese patients have an impaired body image and alteration in self-esteem. An obese patient with a positive body image would be unlikely to seek this surgery unless he or she was experiencing significant comorbidities.

30. A patient has been prescribed orlistat (Xenical) for the treatment of obesity. When providing relevant health education for this patient, the nurse should ensure the patient is aware of what potential adverse effect of treatment? A) Bowel incontinence B) Flatus with oily discharge C) Abdominal pain D) Heat intolerance

B Feedback: Side effects of orlistat include increased frequency of bowel movements, gas with oily discharge, decreased food absorption, decreased bile flow, and decreased absorption of some vitamins. This drug does not cause bowel incontinence, abdominal pain, or heat intolerance.

nurse is teaching client who has barretts esophagus and is scheduled to undergo a EGD. which of the following statement should the nurse include? A) "this procedure is preformed to measure the presence of acid in your esophagus" B) "this procedure can determine how well the lower part of your esophagus works" C) "this procedure is preformed while your under general anesthesia" D) "this procedure will determine if you have colon cancer"

B) "this procedure can determine how well the lower part of your esophagus works"

nurse is caring for a pt who is scheduled to undergo an EGD. the nurse should identify that this procedure is used to do which of the following? A) visualize polyps in the colon B) detect ulceration in the stomach C) identify an obstruction in the biliary tract D) determine presence of free air in the abdomen

B) detect ulceration in the stomach

23. A nurse is providing care for a patient who is postoperative day 2 following gastric surgery. The nurse's assessment should be planned in light of the possibility of what potential complications? Select all that apply. A) Malignant hyperthermia B) Atelectasis C) Pneumonia D) Metabolic imbalances E) Chronic gastritis

B, C, D Feedback: After surgery, the nurse assesses the patient for complications secondary to the surgical intervention, such as pneumonia, atelectasis, or metabolic imbalances resulting from the GI disruption. Malignant hyperthermia is an intraoperative complication. Chronic gastritis is not a surgical complication.

expected assessment findings for client with Peritonitis

BOARD-LIKE ABDOMEN

Expected lab results for pt. with acute kidney injury

BUN 30, urine output of 40ml/3hr, hematocrit 30%

A client is admitted to the health care facility with a diagnosis of a bleeding gastric ulcer. The nurse expects this client's stools to be:

Black and tarry

The nurse is assessing a client with a bleeding gastric ulcer. When examining the client's stool, which of the following characteristics would the nurse be most likely to find?

Black and tarry appearance

5. A nurse is preparing to discharge a patient after recovery from gastric surgery. What is an appropriate discharge outcome for this patient? A) The patient's bowel movements maintain a loose consistency. B) The patient is able to tolerate three large meals a day. C) The patient maintains or gains weight. D) The patient consumes a diet high in calcium.

C Feedback: Expected outcomes for the patient following gastric surgery include ensuring that the patient is maintaining or gaining weight (patient should be weighed daily), experiencing no excessive diarrhea, and tolerating six small meals a day. Patients may require vitamin B12 supplementation by the intramuscular route and do not require a diet excessively rich in calcium.

17. A patient is receiving education about his upcoming Billroth I procedure (gastroduodenostomy). This patient should be informed that he may experience which of the following adverse effects associated with this procedure? A) Persistent feelings of hunger and thirst B) Constipation or bowel incontinence C) Diarrhea and feelings of fullness D) Gastric reflux and belching

C Feedback: Following a Billroth I, the patient may have problems with feelings of fullness, dumping syndrome, and diarrhea. Hunger and thirst, constipation, and gastric reflux are not adverse effects associated with this procedure.

20. A nurse is providing patient education for a patient with peptic ulcer disease secondary to chronic nonsteroidal anti-inflammatory drug (NSAID) use. The patient has recently been prescribed misoprostol (Cytotec). What would the nurse be most accurate in informing the patient about the drug? A) It reduces the stomach's volume of hydrochloric acid B) It increases the speed of gastric emptying C) It protects the stomach's lining D) It increases lower esophageal sphincter pressure

C Feedback: Misoprostol is a synthetic prostaglandin that, like prostaglandin, protects the gastric mucosa. NSAIDs decrease prostaglandin production and predispose the patient to peptic ulceration. Misoprostol does not reduce gastric acidity, improve emptying of the stomach, or increase lower esophageal sphincter pressure.

7. A nurse is completing a health history on a patient whose diagnosis is chronic gastritis. Which of the data should the nurse consider most significantly related to the etiology of the patient's health problem? A) Consumes one or more protein drinks daily. B) Takes over-the-counter antacids frequently throughout the day. C) Smokes one pack of cigarettes daily. D) Reports a history of social drinking on a weekly basis.

C Feedback: Nicotine reduces secretion of pancreatic bicarbonate, which inhibits neutralization of gastric acid and can underlie gastritis. Protein drinks do not result in gastric inflammation. Antacid use is a response to experiencing symptoms of gastritis, not the etiology of gastritis. Alcohol ingestion can lead to gastritis; however, this generally occurs in patients with a history of consumption of alcohol on a daily basis.

21. A nurse is providing anticipator guidance to a patient who is preparing for bariatric surgery. The nurse learns that the patient is anxious about numerous aspects of the surgery. What intervention is most appropriate to alleviate the patient's anxiety? A) Emphasize the fact that bariatric surgery has a low risk of complications. B) Encourage the patient to focus on the benefits of the surgery. C) Facilitate the patient's contact with a support group. D) Obtain an order for a PRN benzodiazepine.

C Feedback: Support groups can be highly beneficial in relieving preoperative and postoperative anxiety and in promoting healthy coping. This is preferable to antianxiety medications. Downplaying the risks of surgery or focusing solely on the benefits is a simplistic and patronizing approach.

15. A patient who underwent gastric banding 3 days ago is having her diet progressed on a daily basis. Following her latest meal, the patient complains of dizziness and palpitations. Inspection reveals that the patient is diaphoretic. What is the nurse's best action? A) Insert a nasogastric tube promptly. B) Reposition the patient supine. C) Monitor the patient closely for further signs of dumping syndrome. D) Assess the patient for signs and symptoms of aspiration.

C Feedback: The patient's symptoms are characteristic of dumping syndrome, which results in a sensation of fullness, weakness, faintness, dizziness, palpitations, diaphoresis, cramping pains, and diarrhea. Aspiration is a less likely cause for the patient's symptoms. Supine positioning will likely exacerbate the symptoms and insertion of an NG tube is contraindicated due to the nature of the patient's surgery.

31. A patient who is obese has been unable to lose weight successfully using lifestyle modifications and has mentioned the possibility of using weight-loss medications. What should the nurse teach the patient about pharmacologic interventions for the treatment of obesity? A) "Weight loss drugs have many side effects, and most doctors think they'll all be off the market in a few years." B) "There used to be a lot of hope that medications would help people lose weight, but it's been shown to be mostly a placebo effect." C) "Medications can be helpful, but few people achieve and maintain their desired weight loss with medications alone." D) "Medications are rapidly become the preferred method of weight loss in people for whom diet and exercise have not worked."

C Feedback: Though antiobesity drugs help some patients lose weight, their use rarely results in loss of more than 10% of total body weight. Patients are consequently unlikely to attain their desired weight through medication alone. They are not predicted to disappear from the market and results are not attributed to a placebo effect.

24. A patient is undergoing diagnostic testing for a tumor of the small intestine. What are the most likely symptoms that prompted the patient to first seek care? A) Hematemesis and persistent sensation of fullness B) Abdominal bloating and recurrent constipation C) Intermittent pain and bloody stool D) Unexplained bowel incontinence and fatty stools

C Feedback: When the patient is symptomatic from a tumor of the small intestine, benign tumors often present with intermittent pain. The next most common presentation is occult bleeding. The other listed signs and symptoms are not normally associated with the presentation of small intestinal tumors.

nurse is caring for a pt with acute pancreatitis. which of the following serum lab values should the nurse anticipate returning to the expected rage within 72hrs after treatment began? A) aldolase B) lipase C) amylase D) lactic dehydrogenase

C) amylase

nurse is caring for pt receiving TPN and just returned to their room following physical therapy. the nurse notes the pump for TPN is turned off. after restarting the pump, the nurse should monitor for which of the following findings? A) HTN B) excessive thirst C) diaphoresis D) fever

C) diaphoresis -possible hypoglycemia

a nurse is providing care dietary teaching to a client who has diverticulitis about preventing acute attacks. which of the following foods should the nurse recommend? A) high in vit C B) low in fat C) high in fiber D) low in cal

C) high in fiber

pt was admitted with bowel obstruction. pt reports severe abdominal pain. which of the findings indicates that possible bowel perforation has occurred? A) elevated BP B) bowel sounds increased in frequency & pitch C) rigid abdomen D) emesis of undigested food

C) rigid abdomen

nurse in ER is caring for a pt who has bleeding esophageal varices. the nurse should anticipate a prescription for which of the following medications? A) famotidine B) esomeprazole C) vasopressin D) omeprazole

C) vasopressin

pt with colorectal cancer receiving chemo asks why blood is drawn for Carcinoembryonic Antigen (CEA) level. response by nurse...

CEA Determines efficacy of chemo - will DECREASE if effective

When talking with a patient about taking metoclopramide (Reglan) to treat GERD, the health care professional should include which of the following instructions? Take the drug once a day in the morning. Stop taking the drug if drowsiness develops. Chew gum or suck on hard candy. Take acetaminophen (Tylenol) for headaches.

Chew gum or suck on hard candy. rationale: Metoclopramide, a prokinetic, can cause anticholinergic effects, including dry mouth. Chewing gum, sucking on hard candy, and sipping water can help minimize this effect.

A patient has a BMI ranger greater than 40 kg/m2. What would this patient's obesity classification be?

Class III

When assessing a client with cirrhosis of the liver, which of the following stool characteristics is the client likely to report? a) Yellow-green b) Clay-colored or whitish c) Blood tinged d) Black and tarry

Clay-colored or whitish Many clients report passing clay-colored or whitish stools as a result of no bile in the gastrointestinal tract. The other stool colors would not be absolute indicators of cirrhosis of the liver but may indicate other GI tract disorders.

A nurse is caring for a client with cholelithiasis. Which sign indicates obstructive jaundice? a) Reduced hematocrit b) Elevated urobilinogen in the urine c) Straw-colored urine d) Clay-colored stools

Clay-colored stools Obstructive jaundice develops when a stone obstructs the flow of bile in the common bile duct. When the flow of bile to the duodenum is blocked, the lack of bile pigments results in a clay-colored stool. In obstructive jaundice, urine tends to be dark amber (not straw-colored) as a result of soluble bilirubin in the urine. Hematocrit levels aren't affected by obstructive jaundice. Because obstructive jaundice prevents bilirubin from reaching the intestine (where it's converted to urobilinogen), the urine contains no urobilinogen.

Which of the following clients is at highest risk for peptic ulcer disease?

Client with blood type O

A client with gastric cancer is having a resection. What is the nursing management priority for this client?

Correcting nutritional deficits

A nurse caring for a patient in a burn treatment center knows to assess for the presence of which of the following types of ulcer about 72 hours post injury?

Curling's

A patient sustained second- and third-degree burns over 30% of the body surface area approximately 72 hours ago. What type of ulcer should the nurse be alert for while caring for this patient?

Curling's ulcer

13. A patient with a peptic ulcer disease has had metronidazole (Flagyl) added to his current medication regimen. What health education related to this medication should the nurse provide? A) Take the medication on an empty stomach. B) Take up to one extra dose per day if stomach pain persists. C) Take at bedtime to mitigate the effects of drowsiness. D) Avoid drinking alcohol while taking the drug.

D Feedback: Alcohol must be avoided when taking Flagyl and the medication should be taken with food. This drug does not cause drowsiness and the dose should not be adjusted by the patient.

32. A patient has been diagnosed with peptic ulcer disease and the nurse is reviewing his prescribed medication regimen with him. What is currently the most commonly used drug regimen for peptic ulcers? A) Bismuth salts, antivirals, and histamine-2 (H2) antagonists B) H2 antagonists, antibiotics, and bicarbonate salts C) Bicarbonate salts, antibiotics, and ZES D) Antibiotics, proton pump inhibitors, and bismuth salts

D Feedback: Currently, the most commonly used therapy for peptic ulcers is a combination of antibiotics, proton pump inhibitors, and bismuth salts that suppress or eradicate H. pylori. H2 receptor antagonists are used to treat NSAID-induced ulcers and other ulcers not associated with H. pylori infection, but they are not the drug of choice. Bicarbonate salts are not used. ZES is the Zollinger-Ellison syndrome and not a drug.

22. A patient has just been diagnosed with acute gastritis after presenting in distress to the emergency department with abdominal symptoms. What would be the nursing care most needed by the patient at this time? A) Teaching the patient about necessary nutritional modification B) Helping the patient weigh treatment options C) Teaching the patient about the etiology of gastritis D) Providing the patient with physical and emotional support

D Feedback: For acute gastritis, the nurse provides physical and emotional support and helps the patient manage the symptoms, which may include nausea, vomiting, heartburn, and fatigue. The scenario describes a newly diagnosed patient; teaching about the etiology of the disease, lifestyle modifications, or various treatment options would be best provided at a later time.

18. A patient has experienced symptoms of dumping syndrome following bariatric surgery. To what physiologic phenomenon does the nurse attribute this syndrome? A) Irritation of the phrenic nerve due to diaphragmatic pressure B) Chronic malabsorption of iron and vitamins A and C C) Reflux of bile into the distal esophagus D) A sudden release of peptides

D Feedback: For many years, it had been theorized that the hypertonic gastric food boluses that quickly transit into the intestines drew extracellular fluid from the circulating blood volume into the small intestines to dilute the high concentration of electrolytes and sugars, resulting in symptoms. Now, it is thought that this rapid transit of the food bolus from the stomach into the small intestines instead causes a rapid and exuberant release of metabolic peptides that are responsible for the symptoms of dumping syndrome. It is not a result of phrenic nerve irritation, malabsorption, or bile reflux.

37. A nurse is caring for a patient hospitalized with an exacerbation of chronic gastritis. What health promotion topic should the nurse emphasize? A) Strategies for maintaining an alkaline gastric environment B) Safe technique for self-suctioning C) Techniques for positioning correctly to promote gastric healing D) Strategies for avoiding irritating foods and beverages

D Feedback: Measures to help relieve pain include instructing the patient to avoid foods and beverages that may be irritating to the gastric mucosa and instructing the patient about the correct use of medications to relieve chronic gastritis. An alkaline gastric environment is neither possible nor desirable. There is no plausible need for self-suctioning. Positioning does not have a significant effect on the presence or absence of gastric healing.

9. A community health nurse is preparing for an initial home visit to a patient discharged following a total gastrectomy for treatment of gastric cancer. What would the nurse anticipate that the plan of care is most likely to include? A) Enteral feeding via gastrostomy tube (G tube) B) Gastrointestinal decompression by nasogastric tube C) Periodic assessment for esophageal distension D) Monthly administration of injections of vitamin B12

D Feedback: Since vitamin B12 is absorbed in the stomach, the patient requires vitamin B12 replacement to prevent pernicious anemia. A gastrectomy precludes the use of a G tube. Since the stomach is absent, a nasogastric tube would not be indicated. As well, this is not possible in the home setting. Since there is no stomach to act as a reservoir and fluids and nutrients are passing directly into the jejunum, distension is unlikely.

4. A nurse is admitting a patient diagnosed with late-stage gastric cancer. The patient's family is distraught and angry that she was not diagnosed earlier in the course of her disease. What factor contributes to the fact that gastric cancer is often detected at a later stage? A) Gastric cancer does not cause signs or symptoms until metastasis has occurred. B) Adherence to screening recommendations for gastric cancer is exceptionally low. C) Early symptoms of gastric cancer are usually attributed to constipation. D) The early symptoms of gastric cancer are usually not alarming or highly unusual.

D Feedback: Symptoms of early gastric cancer, such as pain relieved by antacids, resemble those of benign ulcers and are seldom definitive. Symptoms are rarely a cause for alarm or for detailed diagnostic testing. Symptoms precede metastasis, however, and do not include constipation.

36. A nurse is caring for a patient who has a diagnosis of GI bleed. During shift assessment, the nurse finds the patient to betachycardic and hypotensive, and the patient has an episode of hematemesis while the nurse is in the room. In addition to monitoring the patient's vital signs and level of conscious, what would be a priority nursing action for this patient? A) Place the patient in a prone position. B) Provide the patient with ice water to slow any GI bleeding. C) Prepare for the insertion of an NG tube. D) Notify the physician.

D Feedback: The nurse must always be alert for any indicators of hemorrhagic gastritis, which include hematemesis (vomiting of blood), tachycardia, and hypotension. If these occur, the physician is notified and the patient's vital signs are monitored as the patient's condition warrants. Putting the patient in a prone position could lead to aspiration. Giving ice water is contraindicated as it would stimulate more vomiting.

25. A patient is recovering in the hospital following gastrectomy. The nurse notes that the patient has become increasingly difficult to engage and has had several angry outbursts at various staff members in recent days. The nurse's attempts at therapeutic dialogue have been rebuffed. What is the nurse's most appropriate action? A) Ask the patient's primary care provider to liaise between the nurse and the patient. B) Delegate care of the patient to a colleague. C) Limit contact with the patient in order to provide privacy. D) Make appropriate referrals to services that provide psychosocial support.

D Feedback: The nurse should enlist the services of clergy, psychiatric clinical nurse specialists, psychologists, social workers, and psychiatrists, if needed. This is preferable to delegating care, since the patient has become angry with other care providers as well. It is impractical and inappropriate to expect the primary care provider to act as a liaison. It would be inappropriate and unsafe to simply limit contact with the patient.

nurse is completing a physical assessment on a client with chronic pancreatitis. which of the following is likely the cause of the clients condition? A) high calorie diet B) prior gastro illnesses C) tobacco use D) alcohol use

D) alcohol use

nurse is assessing client who is experiencing perforation of a peptic ulcer. which of the following manifestations should the nurse expect? A) increased blood pressure B) decreased heart rate C) yellowing of the skin D) board like abdomen

D) board like abdomen

nurse is caring for a client who is 2 days postoperative following a gastric bypass. the nurse notes that bowel sounds are present. which of the following foods should the nurse provide at the initial feeding? A) vanilla pudding B) apple juice C) diet ginger ale D) clear liquids

D) clear liquids

a nurse is caring for a client who is dehydrated & receiving continuous tube feeding through a pump at 75ml/hr. when the nurse assess the client at 0800 , which of the following findings requires intervention by the nurse? A) a pitcher full of water is sitting at bedside within reach B) the disposable feeding bag is from the previous day at 1000 with 200ml of feeding C) client is lying on right side with a visible dependent loop in the feeding tube D) the HOB is elevated to 20 degrees

D) the HOB is elevated to 20 degrees

Which of the following would be the most important nursing assessment in a patient diagnosed with ascites? a) Assessment of oral cavity for foul-smelling breath b) Daily weight and measurement of abdominal girth c) Palpation of abdomen for a fluid shift d) Auscultation of abdomen

Daily weight and measurement of abdominal girth Daily measurement and recording of abdominal girth and body weight are essential to assess the progression of ascites and its response to treatment.

A preoperative client scheduled to have an open cholecystectomy says to the nurse, "The doctor said that after surgery, I will have a tube in my nose that goes into my stomach. Why do I need that?" What most common reason for a client having a nasogastric tube in place after abdominal surgery should the nurse include in a response? a) Lavage b) Gavage c) Instillation d) Decompression

Decompression Negative pressure exerted through a tube inserted in the stomach removes secretions and gaseous substances from the stomach, preventing abdominal distention, nausea, and vomiting. This is not the most common purpose of a nasogastric tube after surgery; instillations in a nasogastric tube after surgery are done when necessary to promote patency. This is contraindicated after abdominal surgery until peristalsis returns. This is not the most common purpose of a nasogastric tube after surgery; lavage after surgery may be done to promote hemostasis in the presence of gastric bleeding.

assessment immediately following aParacentesis for treatment of ascites. what indicates procedure was effective

Decreased SOB

Which of the following appears to be a significant factor in the development of gastric cancer?

Diet

A health care professional should question the use of metoclopramide (Reglan) for a patient who is taking which of the following drugs? Digoxin (Lanoxin) Warfarin (Coumadin) Fluvoxamine (Luvox) Allopurinol (Zyloprim)

Digoxin (Lanoxin) rationale: Metoclopramide, a prokinetic drug, can interfere with the absorption of many drugs, including digoxin, diazepam (Valium), tetracycline, and lithium (Lithobid). The patient should not take this drug.

instructions nurse needs to include for a pt Preparing for a colonoscopy

Drink clear liquids for 24 hr prior to procedure and then NPO for 6 hr before

A nurse is caring for a client who underwent a subtotal gastrectomy. To manage dumping syndrome, the nurse should advise the client to:

Drink liquids only between meals.

Clients with Type O blood are at higher risk for which of the following GI disorders?

Duodenal ulcers

Which of the following is the first portion of the small intestine?

Duodenum

discharge teaching for pt with Chronic Cholecystiti. which food choice proves teaching was effective

Eat BANANAS (Low-fat)

pt with new Colostomy concerned about flatus and odor. what food should the nurse recommend?

Eat YOGURT!

What test should the nurse prepare the client for that will locate stones that have collected in the common bile duct? a) Cholecystectomy b) Endoscopic retrograde cholangiopancreatography (ERCP) c) Colonoscopy d) Abdominal x-ray

Endoscopic retrograde cholangiopancreatography (ERCP) ERCP locates stones that have collected in the common bile duct. A colonoscopy will not locate gallstones but only allows visualization of the large intestine. Abdominal x-ray is not a reliable locator of gallstones. A cholecystectomy is the surgical removal of the gallbladder.

A physician suspects that a client has peptic ulcer disease. With which of the following diagnostic procedures would the nurse most likely prepare to assist?

Endoscopy

As a nurse completes the admission assessment of a client admitted for gastric bypass surgery, the client states, "Finally! I'll be thin and able to eat without much concern." How should the nurse intervene?

Evaluate the client's understanding of the procedure.

expected assessment findings for pt with Upper GI bleed

Expect HYPOTENSION

when reviewing lab values for pt with Colorectal Cancer, what findings are expected

Expect decreased hgb (9.1 g/dL)

when reviewing lab values for pt with acute Pancreatitis, what findings are expected

Expect increased serum Amylase

teaching for pt with cirrhosis on prescription for Lactulose

Expect to have 2-3 soft stools per day

PUD - which medication inhibits gastric acid secretion?

Famotidine - H2 Receptor Antagonist

pt with Crohn's Disease, what findings are expected

Fatty, diarrheal stools

The mode of transmission of hepatitis A virus (HAV) includes which of the following? a) Semen b) Fecal-oral c) Saliva d) Blood

Fecal-oral The mode of transmission of hepatitis A virus (HAV) occurs through fecal-oral route, primarily through person to person contact and/or ingestion of fecal contaminated food or water. Hepatitis B virus (HBV) is transmitted primarily through blood. HBV can be found in blood, saliva, semen, and can be transmitted through mucous membranes and breaks in the skin.

The nurse practitioner prescribes an antibiotic for chronic gastritis caused by H. pylori for a patient taking Coumadin. For this patient, which antibiotic should be avoided?

Flagyl

Which type of deficiency results in macrocytic anemia? a) Vitamin K b) Vitamin C c) Vitamin A d) Folic acid

Folic acid Folic acid deficiency results in macrocytic anemia. Vitamin C deficiency results in hemorrhagic lesions of scurvy. Vitamin A deficiency results in night blindness and eye and skin changes. Vitamin K deficiency results in hypoprothrombinemia, which is characterized by spontaneous bleeding and ecchymosis.

Which of the following would be the least important assessment in a patient diagnosed with ascites? a) Foul-smelling breath b) Palpation of abdomen for a fluid shift c) Measurement of abdominal girth d) Weight

Foul-smelling breath Foul-smelling breath would not be considered an important assessment for this patient. Measurement of abdominal girth, weight, and palpation of the abdomen for a fluid shift are all important assessment parameters for the patient diagnosed with ascites.

laxatice complications

GI irritation, rectal burning, toxic levels of magnesium, sodium causes fluid retention, osmotic may cause dehydration. - milk and antacids can destroy enteric coating of bisacodyl

assessment Post-Op Gastrectomy - indication of ACUTE GASTRIC DILATION

HICCUPS - ensure patency of NGT

Which diagnostic test would be used first to evaluate a client with upper GI bleeding?

Hemoglobin and hematocrit

Which type of jaundice is the result of increased destruction of red blood cells? a) Nonobstructive b) Hepatocellular c) Obstructive d) Hemolytic

Hemolytic Hemolytic jaundice is the result of an increased destruction of the red blood cells. Hepatocellular jaundice is caused by the inability of damaged liver cells to clear normal amounts of bilirubin from the blood. Obstructive jaundice resulting from extrahepatic obstruction may be caused by occlusion of the bile duct form a gall stone, inflammatory process, a tumor, or pressure from an enlarged organ. Nonobstructive jaundice occurs with hepatitis.

The nurse identifies which of the following types of jaundice in an adult experiencing a transfusion reaction? a) Hepatocellular b) Nonobstructive c) Obstructive d) Hemolytic

Hemolytic Hemolytic jaundice occurs because, although the liver is functioning normally, it cannot excrete the bilirubin as quickly as it is formed. This type of jaundice is encountered in patients with hemolytic transfusion reactions and other hemolytic disorders. Obstructive jaundice is the result of liver disease. Nonobstructive jaundice occurs with hepatitis. Hepatocellular jaundice is the result of liver disease.

A client with severe peptic ulcer disease has undergone surgery and is several hours postoperative. During assessment, the nurse notes that the client has developed cool skin, tachycardia, and labored breathing; the client also appears to be confused. Which of the following complications has the client most likely developed?

Hemorrhage

A patient has an elevated serum ammonia level and is exhibiting mental status changes. The nurse should suspect which of the following conditions? a) Asterixis b) Cirrhosis c) Hepatic encephalopathy d) Portal hypertension

Hepatic encephalopathy Hepatic encephalopathy is a central nervous system dysfunction resulting from liver disease. It is frequently associated with elevated ammonia levels that produce changes in mental status, altered level of consciousness, and coma. Portal hypertension is an elevated pressure in the portal circulation resulting from obstruction of venous flow into and through the liver. Asterixis is an involuntary flapping movement of the hands associated with metabolic liver dysfunction.

A client and spouse are visiting the clinic. The client recently experienced a seizure and says she has been having difficulty writing. Before the seizure, the client says that for several weeks she was sleeping late into the day but having restlessness and insomnia at night. The client's husband says that he has noticed the client has been moody and slightly confused. Which of the following problems is most consistent with the client's clinical manifestations? a) Esophageal varices b) Hepatic encephalopathy c) Portal hypertension d) Hepatitis C

Hepatic encephalopathy The earliest symptoms of hepatic encephalopathy include minor mental changes and motor disturbances. The client appears slightly confused and unkempt and has alterations in mood and sleep patterns. The client tends to sleep during the day and have restlessness and insomnia at night. As hepatic encephalopathy progresses, the client may become difficult to awaken and completely disoriented with respect to time and place. With further progression, the client lapses into frank coma and may have seizures. Simple tasks, such as handwriting, become difficult.

A nurse is preparing a presentation for a local community group about hepatitis. Which of the following would the nurse include? a) Infection with hepatitis G is similar to hepatitis A. b) Hepatitis B is transmitted primarily by the oral-fecal route. c) Hepatitis C increases a person's risk for liver cancer. d) Hepatitis A is frequently spread by sexual contact.

Hepatitis C increases a person's risk for liver cancer. Infection with hepatitis C increases the risk of a person developing hepatic (liver) cancer. Hepatitis A is transmitted primarily by the oral-fecal route; hepatitis B is frequently spread by sexual contact and infected blood. Hepatitis E is similar to hepatitis A whereas hepatitis G is similar to hepatitis C.

Patients diagnosed with esophageal varices are at risk for hemorrhagic shock. Which of the following is a sign of potential hypovolemia? a) Bradycardia b) Hypotension c) Polyuria d) Warm moist skin

Hypotension Signs of potential hypovolemia include cool, clammy skin, tachycardia, decreased blood pressure, and decreased urine output.

A health care professional should question the use of diphenoxylate/atropine (Lomotil) for a patient who has which of the following disorders? Inflammatory bowel disease Thrombophlebitis Agranulocytosis Immunization with a live virus

IBS rationale: Diphenoxylate is an opioid, which can cause the severe complication of toxic megacolon in patients who have inflammatory bowel disease. This disorder is a contraindication for the use of the drug.

aminosalicylates: mesalamine, olsalazine, hydrocortisone, azathioprine, infliximab, mentronidazole

IBS, crohns, ulcerative colitis Adverse: blood disorders, nausea, cramps, rash, arthralgia

Which of the following dietary guidelines should be followed following bariatric surgery? Select all that apply.

Include two protein snacks per day. Eat slowly. Eat three meals per day.

to Prevent Dumping Syndrome post-op following gastrectomy, which food should be included in diet

Increase proteins - EGGS

The client has been taking famotidine (Pepcid) at home. The nurse prepares a teaching plan for the client indicating that the medication acts primarily to achieve which of the following?

Inhibit gastric acid secretions.

The nurse is caring for a client who underwent a subtotal gastrectomy 36 hours ago. The client has a nasogastric (NG) tube. The nurse knows to do the following with the NG:

Irrigate the NG tube gently with normal saline if ordered.

pt with Hepatitis B, what findings are expected

JOINT PAIN

The nurse is administering medications to a patient that has elevated ammonia due to cirrhosis of the liver. What medication will the nurse give to detoxify ammonium and to act as an osmotic agent? a) Spironolactone (Aldactone) b) Cholestyramine (Questran) c) Kanamycin (Kantrex) d) Lactulose (Cephulac)

Lactulose (Cephulac) Lactulose (Cephulac) is administered to detoxify ammonium and to act as an osmotic agent, drawing water into the bowel, which causes diarrhea in some clients. Potassium-sparing diuretics such as spironolactone (Aldactone) are used to treat ascites. Cholestyramine (Questran) is a bile acid sequestrant and reduces pruritus. Kanamycin (Kantrex) decreases intestinal bacteria and decreases ammonia but does not act as an osmotic agent.

A client is being prepared to undergo laboratory and diagnostic testing to confirm the diagnosis of cirrhosis. Which test would the nurse expect to be used to provide definitive confirmation of the disorder? a) Magnetic resonance imaging b) Liver biopsy c) Coagulation studies d) Radioisotope liver scan

Liver biopsy A liver biopsy which reveals hepatic fibrosis is the most conclusive diagnostic procedure. Coagulation studies provide information about liver function but do not definitively confirm the diagnosis of cirrhosis. Magnetic resonance imaging and radioisotope liver scan help to support the diagnosis but do not confirm it. These tests provide information about the liver's enlarged size, nodular configuration, and distorted blood flow.

A physician orders spironolactone (Aldactone), 50 mg by mouth four times daily, for a client with fluid retention caused by cirrhosis. Which finding indicates that the drug is producing a therapeutic effect? a) Blood pH of 7.25 b) Loss of 2.2 lb (1 kg) in 24 hours c) Serum sodium level of 135 mEq/L d) Serum potassium level of 3.5 mEq/L

Loss of 2.2 lb (1 kg) in 24 hours Daily weight measurement is the most accurate indicator of fluid status; a loss of 2.2 lb (1 kg) indicates loss of 1 L of fluid. Because spironolactone is a diuretic, weight loss is the best indicator of its effectiveness. This client's serum potassium and sodium levels are normal. A blood pH of 7.25 indicates acidosis, an adverse reaction to spironolactone.

A patient with cirrhosis has a massive hemorrhage from esophageal varices. Balloon tamponade therapy is used temporarily to control hemorrhage and stabilize the patient. In planning care, the nurse gives the highest priority to which of the following goals? a) Maintaining the airway b) Relieving the patient's anxiety c) Controlling bleeding d) Maintaining fluid volume

Maintaining the airway Esophageal varices are almost always caused by portal hypertension, which results from obstruction of the portal circulation within the damaged liver. Maintaining the airway is the highest priority because oxygenation is essential for life. The airway is compromised by possible displacement of the tube and the inflated balloon into the oropharynx, which can cause life-threatening obstruction of the airway and asphyxiation.

When performing a physical examination on a client with cirrhosis, a nurse notices that the client's abdomen is enlarged. Which of the following interventions should the nurse consider? a) Measure abdominal girth according to a set routine. b) Report the condition to the physician immediately. c) Provide the client with nonprescription laxatives. d) Ask the client about food intake.

Measure abdominal girth according to a set routine. If the abdomen appears enlarged, the nurse measures it according to a set routine. The nurse reports any change in mental status or signs of gastrointestinal bleeding immediately. It is not essential for the client to take laxatives unless prescribed. The client's food intake does not affect the size of the abdomen in case of cirrhosis.

interventions to include in care plan for pt with Cirrhosis and Ascites

Measure abdominal girth daily

A nurse educator is providing an in-service to a group of nurses working on a medical floor that specializes in liver disorders. Which of the following is an important education topic regarding ingestion of medications? a) The need for more frequently divided doses b) The need for increased drug dosages c) Medications will no longer be effective in clients with liver disease. d) Metabolism of medications

Metabolism of medications Careful evaluation of the client's response to drug therapy is important because the malfunctioning liver cannot metabolize many substances.

Which of the following is a true statement regarding gastric cancer?

Most patients are asymptomatic during the early stage of the disease.

when reviewing prescriptions for pt with Campylobacter Enteritis which should the nurse clarify with provider

NO Milk of Mag - increases GI motility and increases risk of electrolyte imbalances

A nurse is caring for a client with active upper GI bleeding. What is the appropriate diet for this client during the first 24 hours after admission?

NPO

Ginger root

Nausea, vertigo, GI spasms, anti-inflammatory Adverse: careful w/ pregancy, interacts w/ coagulation

A client undergoes total gastrectomy. Several hours after surgery, the nurse notes that the client's nasogastric (NG) tube has stopped draining. How should the nurse respond?

Notify the physician

A client is admitted to the hospital with acute hemorrhage from esophageal varices. What medication should the nurse anticipate administering that will reduce pressure in the portal venous system and control esophageal bleeding? a) Vasopressin (Pitressin) b) Vitamin K c) Epinephrine d) Octreotide (Sandostatin)

Octreotide (Sandostatin) Acute hemorrhage from esophageal varices is lifethreatening. Resuscitative measures include administration of IV fluids and blood products. IV octreotide (Sandostatin) is started as soon as possible. Sandostatin is preferred because of fewer side effects. Octreotide reduces pressure in the portal venous system and is preferred to the previously used agents, vasopressin (Pitressin) or terlipressin. Vitamin K promotes blood coagulation in bleeding conditions, resulting from liver disease.

Which of the following medications is classified as a proton pump inhibitor (PPI)?

Omeprazole

A nurse practitioner prescribes drug therapy for a patient with peptic ulcer disease. Choose the drug that can be used for 4 weeks and has a 90% chance of healing the ulcer.

Omeprazole (Prilosec)

Which of the following represents the medication classification of a proton (gastric acid) pump inhibitor?

Omeprazole (Prilosec)

expected assessment finding for client with Appendicitis

Oral temp 101.1, Nausea/Vommiting, RLQ pain

first action nurse should take for admission who has acute Pancreatitis

PAIN SCALE - first identify level of pain

To reduce risk of injury for a patient with liver disease, what initial measure can the nurse implement? a) Raise all four side rails on the bed b) Prevent visitors, so as not to agitate the patient c) Pad the side rails on the bed d) Apply soft wrist restraints

Pad the side rails on the bed Padding the side rails can reduce injury if the patient becomes agitated or restless. Restraints would not be an initial measure to implement. Four side rails are considered a restraint and this would not be an initial measure to implement. Family and friends most generally assist in calming a patient.

The nurse recognizes that the patient diagnosed with a duodenal ulcer will likely experience

Pain 2 to 3 hours after a meal.

pt with DUODENAL Ulcer, what findings are expected

Pain occurs 1.5-3 hr after meals and at NIGHT

A nurse is gathering equipment and preparing to assist with a sterile bedside procedure to withdraw fluid from a patient's abdomen. The procedure tray contains the following equipment: trocar, syringe, needles, and drainage tube. The patient is placed in a high Fowler's position and a BP cuff is secured around the arm in preparation for which of the following procedures? a) Liver biopsy b) Dialysis c) Paracentesis d) Abdominal ultrasound

Paracentesis Paracentesis is the removal of fluid (ascites) from the peritoneal cavity through a puncture or a small surgical incision through the abdominal wall under sterile conditions. Paracentesis may be used to withdraw ascitic fluid if the fluid accumulation is causing cardiorespiratory compromise.

Louisa Collins, a 52-year-old high school science teacher, is your client on the medical-surgical floor at the hospital where you practice nursing. She is undergoing diagnostics to determine what is causing her liver disorder. As you discuss her condition, Louisa asks questions pertaining to liver physiology and cellular function. Which liver cells perform most of the liver's metabolic functions? a) Parenchymal cells b) Canaliculi cells c) Kupffer cells d) Islets of Langerhans

Parenchymal cells

Louisa Collins, a 52-year-old high school science teacher, is your client on the medical-surgical floor at the hospital where you practice nursing. She is undergoing diagnostics to determine what is causing her liver disorder. As you discuss her condition, Louisa asks questions pertaining to liver physiology and cellular function. Which liver cells perform most of the liver's metabolic functions? a) Parenchymal cells b) Canaliculi cells c) Kupffer cells d) Islets of Langerhans

Parenchymal cells The cellular constituents of the liver include the hepatic parenchymal cells (hepatocytes), which perform most of the liver's metabolic functions. The parenchymal cells perform most of the liver's metabolic functions.

Which of the following are classified as a histamine-2 receptor antagonist?

Pepcid

A nurse is caring for a client who is undergoing a diagnostic workup for a suspected GI problem. The client reports gnawing epigastric pain following meals and heartburn. The nurse suspects the client has:

Peptic ulcer disease.

The most common cause of esophageal varices includes which of the following? a) Portal hypertension b) Jaundice c) Ascites d) Asterixis

Portal hypertension Esophageal varices are almost always caused by portal hypertension, which results from obstruction of the portal circulation within the damaged liver. Jaundice occurs when the bilirubin concentration in the blood is abnormally elevated. Ascites results from circulatory changes within the diseased liver. Asterixis is an involuntary flapping movement of the hands associated with metabolic liver dysfunction.

An elderly homeless client with a lengthy history of alcohol addiction is visiting the health clinic where you work. He has worsening jaundice. After diagnostic testing is complete, the physician returns a diagnosis of cirrhosis. The nurse begins client education about this condition. What would the nurse emphasize as the principal goal of cirrhosis therapy? a) Restoring fat-soluble vitamin absorption b) Curing the illness c) Preserving liver function d) Increasing alcohol toleration

Preserving liver function The principal aim of therapy is to prevent further deterioration by abolishing underlying causes and preserving what liver function remains. The principal aim of therapy is to prevent further deterioration.

The nurse is caring for a client with cirrhosis. Which assessment findings indicate that the client has deficient vitamin K absorption caused by this hepatic disease? a) Gynecomastia and testicular atrophy b) Ascites and orthopnea c) Purpura and petechiae d) Dyspnea and fatigue

Purpura and petechiae A hepatic disorder, such as cirrhosis, may disrupt the liver's normal use of vitamin K to produce prothrombin (a clotting factor). Consequently, the nurse should monitor the client for signs of bleeding, including purpura and petechiae. Dyspnea and fatigue suggest anemia. Ascites and orthopnea are unrelated to vitamin K absorption. Gynecomastia and testicular atrophy result from decreased estrogen metabolism by the diseased liver.

Connie, a 60-year-old retired financial planner, is recently diagnosed with carcinoma of the pancreas. She has just met with her surgeon and feels overwhelmed by all the information she was given. She tells you that she is having the head of the pancreas removed; additionally, the surgeon is also removing the duodenum and stomach and redirecting the flow of secretions from the stomach, gallbladder, and pancreas into the middle section of the small intestine. What procedure is Connie having performed? a) Distal pancreatectomy b) Total pancreatectomy c) Cholecystojejunostomy d) Radical pancreatoduodenectomy

Radical pancreatoduodenectomy This surgical procedure involves removing the head of the pancreas, resecting the duodenum and stomach, and redirecting the flow of secretions from the stomach, gallbladder, and pancreas into the jejunum. This surgical procedure is a rerouting of the pancreatic and biliary drainage systems, which may be done to relieve obstructive jaundice. This measure is considered palliative only. A pancreatectomy is the surgical removal of the pancreas. A pancreatectomy may be total, in which case the entire organ is removed, usually along with the spleen, gallbladder, common bile duct, and portions of the small intestine and stomach. A distal pancreatectomy is a surgical procedure to remove the bottom half of the pancreas.

Which of the following liver function studies is used to show the size of the liver and hepatic blood flow and obstruction? a) Radioisotope liver scan b) Magnetic resonance imaging (MRI) c) Electroencephalogram (EEG) d) Angiography

Radioisotope liver scan A radioisotope liver scan assesses liver size and hepatic blood flow and obstruction. An MRI is used to identify normal structures and abnormalities of the liver and biliary tree. Angiography is used to visualize hepatic circulation and detect the presence and nature of hepatic masses. An EEG is used to detect abnormalities that occur with hepatic coma.

A client with cirrhosis has portal hypertension, which is causing esophageal varices. What is the goal of the interventions that the nurse will provide? a) Cure the cirrhosis. b) Reduce fluid accumulation and venous pressure. c) Treat the esophageal varices. d) Promote optimal neurologic function.

Reduce fluid accumulation and venous pressure. Methods of treating portal hypertension aim to reduce fluid accumulation and venous pressure. There is no cure for cirrhosis; treating the esophageal varices is only a small portion of the overall objective. Promoting optimal neurologic function will not reduce portal hypertension.

The nurse advises the patient who has just been diagnosed with acute gastritis to:

Refrain from food until the GI symptoms subside.

Which of the following is the most successful treatment for gastric cancer?

Removal of the tumor

when reviewing labs for pt with Cirrhosis, what lab value should be reported to provider

Report Ammonia 180 mcg/dL = ENCEPHALOPATHY

assessing a client with Cirrhosis, what is priority to be reported to the provider

Report bloody stools - at risk for hemorrhaging and indicates GI bleed

following an Ileostomy, pt should be taught to report what finding after discharge

Report purple stoma

The nurse is providing care to a patient with gross ascites who is maintaining a position of comfort in the high semi-Fowler's position. What is the nurse's priority assessment of this patient? a) Urinary output related to increased sodium retention b) Peripheral vascular assessment related to immobility c) Skin assessment related to increase in bile salts d) Respiratory assessment related to increased thoracic pressure

Respiratory assessment related to increased thoracic pressure If a patient with ascites from liver dysfunction is hospitalized, nursing measures include assessment and documentation of intake and output (I&O;), abdominal girth, and daily weight to assess fluid status. The nurse also closely monitors the respiratory status because large volumes of ascites can compress the thoracic cavity and inhibit adequate lung expansion. The nurse monitors serum ammonia, creatinine, and electrolyte levels to assess electrolyte balance, response to therapy, and indications of encephalopathy.

After undergoing a liver biopsy, a client should be placed in which position? a) Right lateral decubitus position b) Semi-Fowler's position c) Supine position d) Prone position

Right lateral decubitus position After a liver biopsy, the client is placed on the right side (right lateral decubitus position) to exert pressure on the liver and prevent bleeding. Semi-Fowler's position and the supine and prone positions wouldn't achieve this goal.

In what location would the nurse palpate for the liver? a) Right upper quadrant b) Left upper quadrant c) Left lower quadrant d) Right lower quadrant

Right upper quadrant The liver may be palpable in the right upper quadrant. A palpable liver presents as a firm, sharp ridge with a smooth surface.

Patients with chronic liver dysfunction have problems with insufficient vitamin intake. Which of the following may occur as a result of vitamin C deficiency? a) Hypoprothrombinemia b) Scurvy c) Night blindness d) Beriberi

Scurvy Scurvy may result from a vitamin C deficiency. Night blindness, hypoprothrombinemia, and beriberi do not result from a vitamin C deficiency.

A patient has a Class II classification of obesity. What level of health risk does this pose for the patient?

Severe Risk

Which of the following medications used for the treatment of obesity prevents the reuptake of serotonin and norepinephrine?

Sibutramine hydrochloride (Meridia)

A patient with hepatic cirrhosis questions the nurse about the possible use of an herbal supplement—milk thistle—to help heal the liver. Which of the following would be the most appropriate response from the nurse? a) Silymarin from milk thistle has anti-inflammatory and antioxidant properties that may have beneficial effects, especially in hepatitis.However, you should always notify your primary care provider of any herbal remedies being used so drug interactions can be evaluated. b) You can use milk thistle instead of the medications you have been prescribed. c) Herbal supplements are approved by the FDA, so there should be no problem with their usage if you check with your primary care provider. d) You should not use herbal supplements in conjunction with medical treatment.

Silymarin from milk thistle has anti-inflammatory and antioxidant properties that may have beneficial effects, especially in hepatitis.However, you should always notify your primary care provider of any herbal remedies being used so drug interactions can be evaluated. Many patients who have end-stage liver disease (ESLD) with cirrhosis use the herb milk thistle (Silybum marianum) to treat jaundice and other symptoms. This herb has been used for centuries because of its healing and regenerative properties for liver disease. Silymarin from milk thistle has anti-inflammatory and antioxidant properties that may have beneficial effects, especially in hepatitis. The natural compound SAM-e (s-adenosylmethionine) may improve outcomes in liver disease by improving liver function, possibly through enhancing antioxidant function. Herbal supplements are used in conjunction with medical treatment and medications. Herbal supplements are not approved by the FDA. Their usage should be discussed with the primary care provider to evaluate their effectiveness and interactions with other treatment regimens.

A client is recovering from gastric surgery. Toward what goal should the nurse progress the client's enteral intake?

Six small meals daily with 120 mL fluid between meals

The nurse is concerned about potassium loss when a diuretic is prescribed for a patient with ascites and edema. What diuretic may be ordered that spares potassium and prevents hypokalemia? a) Bumetanide (Bumex) b) Furosemide (Lasix) c) Acetazolamide (Diamox) d) Spironolactone (Aldactone)

Spironolactone (Aldactone) Potassium-sparing diuretic agents such as spironolactone or triamterene (Dyrenium) may be indicated to decrease ascites, if present; these diuretics are preferred because they minimize the fluid and electrolyte changes commonly seen with other agents.

Why should total parental nutrition (TPN) be used cautiously in clients with pancreatitis? a) Such clients can digest high-fat foods. b) Such clients are at risk for gallbladder contraction. c) Such clients cannot tolerate high-glucose concentration. d) Such clients are at risk for hepatic encephalopathy.

Such clients cannot tolerate high-glucose concentration. Total parental nutrition (TPN) is used carefully in clients with pancreatitis because some clients cannot tolerate a high-glucose concentration even with insulin coverage. Intake of coffee increases the risk for gallbladder contraction, whereas intake of high protein increases risk for hepatic encephalopathy in clients with cirrhosis. Patients with pancreatitis should not be given high-fat foods because they are difficult to digest.

A nurse is caring for a client with cirrhosis. The nurse assesses the client at noon and discovers that the client is difficult to arouse and has an elevated serum ammonia level. The nurse should suspect which situation? a) The client didn't take his morning dose of lactulose (Cephulac). b) The client's hepatic function is decreasing. c) The client is avoiding the nurse. d) The client is relaxed and not in pain.

The client's hepatic function is decreasing. The decreased level of consciousness caused by an increased serum ammonia level indicates hepatic dysfunction. If the client didn't take his morning dose of lactulose, he wouldn't have elevated ammonia levels and decreased level of consciousness this soon. These assessment findings don't indicate that the client is relaxed or avoiding the nurse.

A nurse is caring for a client who had gastric bypass surgery 2 days ago. Which assessment finding requires immediate intervention?

The client's right lower leg is red and swollen.

Ammonia, the major etiologic factor in the development of encephalopathy, inhibits neurotransmission. Increased levels of ammonia are damaging to the body. The largest source of ammonia is from: a) The digestion of dietary and blood proteins. b) Excess potassium loss subsequent to prolonged use of diuretics. c) Excessive diuresis and dehydration. d) Severe infections and high fevers.

The digestion of dietary and blood proteins. Circumstances that increase serum ammonia levels tend to aggravate or precipitate hepatic encephalopathy. The largest source of ammonia is the enzymatic and bacterial digestion of dietary and blood proteins in the GI tract. Ammonia from these sources increases as a result of GI bleeding (ie, bleeding esophageal varices, chronic GI bleeding), a high-protein diet, bacterial infection, or uremia.

A patient with bleeding esophageal varices has had pharmacologic therapy with Octreotide (Sandostatin) and endoscopic therapy with esophageal varices banding, but the patient has continued to have bleeding. What procedure that will lower portal pressure does the nurse prepare the patient for? a) Vasopressin (Pitressin) b) Transjugular intrahepatic portosystemic shunting (TIPS) c) Balloon tamponade d) Sclerotherapy

Transjugular intrahepatic portosystemic shunting (TIPS) A TIPS procedure (see Fig. 49-8) is indicated for the treatment of an acute episode of uncontrolled variceal bleeding refractory to pharmacologic or endoscopic therapy. In 10% to 20% of patients for whom urgent band ligation or sclerotherapy and medications are not successful in eradicating bleeding, a TIPS procedure can effectively control acute variceal hemorrhage by rapidly lowering portal pressure.

The nurse is caring for a patient with ascites due to cirrhosis of the liver. What position does the nurse understand will activate the renin-angiotensin aldosterone and sympathetic nervous system and decrease responsiveness to diuretic therapy? a) Prone b) Left-lateral Sims' c) Upright d) Supine

Upright In patients with ascites, an upright posture is associated with activation of the renin-angiotensin-aldosterone system and sympathetic nervous system (Porth & Matfin, 2009). This causes reduced renal glomerular filtration and sodium excretion and a decreased response to loop diuretics.

pt with Ulcerative Colitis who has experienced several exacerbation in the past 3 yrs. which instructions should be included to minimize risk of exacerbations

Use progressive relaxation techniques, arrange activities to allow for daily rest periods, and restrict intake of carbonated beverages; NO increase in fiber; NO dairy

Which of the following is the most effective strategy to prevent hepatitis B infection? a) Vaccine b) Barrier protection during intercourse c) Covering open sores d) Avoid sharing toothbrushes

Vaccine The most effective strategy to prevent hepatitis B infection is through vaccination. Recommendations to prevent transmission of hepatitis B include vaccination of sexual contacts of individuals with chronic hepatitis, use of barrier protection during sexual intercourse, avoidance of sharing toothbrushes, razors with others, and covering open sores or skin lesions.

A nurse is providing care for a client recovering from gastric bypass surgery. During assessment, the client exhibits pallor, perspiration, palpitations, headache, and feelings of warmth, dizziness, and drowsiness. The client reports eating 90 minutes ago. The nurse suspects:

Vasomotor symptoms associated with dumping syndrome

A client is actively bleeding from esophageal varices. Which of the following medications would the nurse most expect to be administered to this client? a) Spironolactone (Aldactone) b) Propranolol (Inderal) c) Lactulose (Cephulac) d) Vasopressin (Pitressin)

Vasopressin (Pitressin) In an actively bleeding client, medications are administered initially because they can be obtained and administered quicker than other therapies. Vasopressin (Pitressin) may be the initial mode of therapy in urgent situations, because it produces constriction of the splanchnic arterial bed and decreases portal pressure. Propranolol (Inderal) and nadolol (Corgard), beta-blocking agents that decrease portal pressure, are the most common medications used both to prevent a first bleeding episode in clients with known varices and to prevent rebleeding. Beta-blockers should not be used in acute variceal hemorrhage, but they are effective prophylaxis against such an episode. Spironolactone (Aldactone), an aldosterone-blocking agent, is most often the first-line therapy in clients with ascites from cirrhosis. Lactulose (Cephulac) is administered to reduce serum ammonia levels in clients with hepatic encephalopathy.

Which of the following medications is used to decrease portal pressure, halting bleeding of esophageal varices? a) Vasopressin (Pitressin) b) Spironolactone (Aldactone) c) Cimetidine (Tagamet) d) Nitroglycerin

Vasopressin (Pitressin) Vasopressin may be the initial therapy for esophageal varices, because it constricts the splanchnic arterial bed and decreases portal hypertension. Nitroglycerin has been used to prevent the side effects of vasopressin. Aldactone and Tagamet do not decrease portal hypertension.

A client with severe and chronic liver disease is showing manifestations related to inadequate vitamin intake and metabolism. He reports difficulty driving at night because he cannot see well. Which of the following vitamins is most likely deficient for this client? a) Thiamine b) Riboflavin c) Vitamin K d) Vitamin A

Vitamin A Problems common to clients with severe chronic liver dysfunction result from inadequate intake of sufficient vitamins. Vitamin A deficiency results in night blindness and eye and skin changes. Thiamine deficiency can lead to beriberi, polyneuritis, and Wernicke-Korsakoff psychosis. Riboflavin deficiency results in characteristic skin and mucous membrane lesions. Vitamin K deficiency can cause hypoprothrombinemia, characterized by spontaneous bleeding and ecchymoses.

A client is admitted to the hospital with an exacerbation of his chronic gastritis. When assessing his nutritional status, the nurse should expect a deficiency in:

Vitamin B-12

A client with carcinoma of the head of the pancreas is scheduled for surgery. Which of the following should a nurse administer to the client before surgery? a) Vitamin B b) Vitamin K c) Oral bile acids d) Potassium

Vitamin K Clients with carcinoma of the head of the pancreas typically require vitamin K before surgery to correct a prothrombin deficiency. Potassium would be given only if the client's serum potassium levels were low. Oral bile acids are not prescribed for a client with carcinoma of the head of the pancreas; they are given to dissolve gallstones. Vitamin B has no implications in the surgery.

A patient with severe chronic liver dysfunction comes to the clinic with bleeding of the gums and blood in the stool. What vitamin deficiency does the nurse suspect the patient may be experiencing? a) Vitamin K deficiency b) Riboflavin deficiency c) Vitamin A deficiency d) Folic acid deficiency

Vitamin K deficiency Vitamin A deficiency results in night blindness and eye and skin changes. Thiamine deficiency leads to beriberi, polyneuritis, and Wernicke-Korsakoff psychosis. Riboflavin deficiency results in characteristic skin and mucous membrane lesions. Pyridoxine deficiency results in skin and mucous membrane lesions and neurologic changes. Vitamin C deficiency results in the hemorrhagic lesions of scurvy. Vitamin K deficiency results in hypoprothrombinemia, characterized by spontaneous bleeding and ecchymoses. Folic acid deficiency results in macrocytic anemia.

pt in ED w/ costovertebral angle tenderness, N/V. what lab values need to be reported?

WBC 15,000

Which of the following indicates an overdose of lactulose? a) Hypoactive bowel sounds b) Constipation c) Fecal impaction d) Watery diarrhea

Watery diarrhea The patient receiving lactulose is monitored closely for the development of watery diarrheal stool, which indicates a medication overdose.

which client should be monitored for nephrotoxicity

a client receiving gentamicin for treatment of wound infection

hemodialysis is contraindicated for what patient

a client who cant receieve anticoagulants

what finding for a client who received hemodialysis puts the client at risk for seizures

a rapid decrease in fluid

A health care professional is caring for a patient who is about to begin taking alosetron (Lotronex) to treat irritable bowel syndrome. The health care professional should tell the patient to report which of the following adverse effects? Headache Drowsiness Abdominal pain Sore throat

abdominal pain rationale: Alosetron, a serotonin 5-HT3 receptor antagonist, can cause ischemic colitis. The health care professional should tell the patient to report abdominal pain, bloody diarrhea, or rectal bleeding, and to stop taking the drug if these symptoms occur.

medications for IBS: alosetron, only for female w/ IBS lasting more than 6m

adverse: constipation, effective in 1-4 weeks

meds for IBS w/ constipation: lubiprostone

adverse: diarrhea, nausea, take w/ food

folic acid

alcohol use, pregnancy anemia:

A primary care provider who is considering the various pharmacologic options for a patient who has peptic ulcer disease should understand that which of the following drugs require monitoring of the patient's phosphorus levels? Omeprazole (Prilosec) Aluminum hydroxide (Amphojel) Sucralfate (Carafate) Ranitidine (Zantac)

aluminum hydroxide rationale: Antacids that contain aluminum, such as aluminum hydroxide, can cause hypophosphatemia because of aluminum's ability to bind with phosphate and decrease its absorption. The drug requires monitoring of phosphorus levels.

A health care professional should question the use of dimenhydrinate for a patient who has which of the following disorders? Hyperthyroidism Angle-closure glaucoma Hypertension Diabetes mellitus

angle-closure glaucoma rationale: Dimenhydrinate, an antihistamine, is inappropriate for patients who have angle-closure glaucoma because it has anticholinergic properties. Anticholinergic drugs can increase intraocular pressure, making angle-closure glaucoma a contraindication for the use of the drug.

scopolamine

anitcholinergic; motion sickness. PO, topical , subQ

A primary care provider should use caution when prescribing bisacodyl (Dulcolax) for a patient who has anorexia nervosa. myelosuppression. hypomagnesemia diabetes mellitus.

anorexia nervosa. rationale: Bisacodyl, a stimulant laxative, requires caution with patients who have an eating disorder because of the risk of laxative abuse. Health care professionals should monitor intake and output carefully and discourage long-term use.

A health care professional is caring for a patient who is about to begin taking ranitidine (Zantac) to treat GERD. The health care professional should tell the patient to take certain precautions when taking which of the following over-the-counter drugs? Ginkgo biloba antidiarrheals st. johns wort antacids

antacids rationale: Antacids can decrease the absorption of ranitidine, a histamine2-receptor antagonist. The health care professional should tell the patient to allow at least 1 hr between taking ranitidine and taking an antacid.

dimenhydrinate

antihistamine: treats motion sickness, PO, IM IV

what action should be taken by the nurse for Pt scheduled for ESWL for urthiasis

apply electrodes for cardiac monitoring

A primary care provider should use caution when prescribing sulfasalazine (Azulfidine) for a patient who has pancreatitis. aspirin sensitivity. bronchitis. GERD.

aspirin Rationale: Patients who have pancreatitis may take sulfasalazine, a 5-aminosalicylate. Pancreatitis is a contraindication for the use of azathioprine (Imuran), another GI-system drug. aspirin sensitivity. Any sensitivity to salicylates, sulfonamides, or trimethoprim (Primsol) is a contraindication for the use of sulfasalazine, a 5-aminosalicylate. This is because intestinal bacteria metabolize the drug into 5-aminosalicylic acid, a salicylate. Aspirin is also a salicylate.

priority nursing assessment for client post-op following nephrectomy

blood pressure

A health care professional is caring for a patient who is about to begin taking azathioprine (Imuran) to treat inflammatory bowel disease. The health care professional should tell the patient to report which of the following adverse effects? (Select all that apply.) Impotence Bruising Jaundice Sore throat Nausea

bruising jaundice sore throat nausea Impotence is incorrect. Azathioprine, an immunosuppressant, is unlikely to cause impotence. Cimetidine (Tagamet) is a GI-system drug that can cause this effect. Bruising is correct. Azathioprine can cause thrombocytopenia. The health care professional should monitor platelet count, and tell the patient to report easy or unexplained bruising or bleeding. Jaundice is correct. Azathioprine can cause hepatitis. The health care professional should monitor liver function tests, and tell the patient to report abdominal pain or jaundice. Sore throat is correct. Azathioprine can cause agranulocytosis. The health care professional should tell the patient to report sore throat or fatigue. Nausea is correct. Azathioprine can cause nausea, vomiting, and anorexia. The health care professional should tell the patient to take azathioprine with food to minimize GI effects.

ondansetron

chemo, radiation, postop emesis. PO and IV adverse: headache, dizziness, diarrhea

inflammatory bowel disease

chronic inflammation of part or all of the digestive tract

A client is admitted for suspected GI disease. Assessment data reveal muscle wasting, a decrease in chest and axillary hair, and increased bleeding tendency. The nurse suspects the client has: a) peptic ulcer disease. b) appendicitis. c) cholelithiasis. d) cirrhosis.

cirrhosis. Muscle wasting, a decrease in chest and axillary hair, and increased bleeding tendencies are all symptoms of cirrhosis. The client may also have mild fever, edema, abdominal pain, and an enlarged liver. Clients with peptic ulcer disease complain of a dull, gnawing epigastric pain that's relieved by eating. Appendicitis is characterized by a periumbilical pain that moves to the right lower quadrant and rebound tenderness. Cholelithiasis is characterized by severe abdominal pain that presents several hours after a large meal.

chronic kidney disease, completed 3rd PD, what finding should be reported to provider?

cloudy dialysate effluent

prokinetic agents: metoclopramide

controls nausea and vomiting by blocking dopamine and serotonin receptors in the CTZ, incrase GI motility- N/V and GERD. Adverse: EPS, sedation, diarrhea

expected finding acute glomerulonephritis

dark urine

bulk-forming; psyllium

decrease diarrhea, control BM for ileostomy, promote defecation in older adults

antidiarrheals: diphenoxylate plus atropine, loperamide, paregoric

decrease intestinal motility, diphen doesnt affect CNS at recommneded doses but at high can- so atropine added for unpleasant effects

Vitamin B12

deficiency affects blood cells, Adverse: hypokalemia, folic acid can mask deficiency,

priority intervention for client the night before a scheduled intravenous urography

determine if the client is allergic to iodine of shellfish

antiemetic med list

dexamethasone, aprepitant, granisetron, prochlorperazine, metoclopramide, promethazine, dronabinol, scopolamine, dimenhydrinate, hydroxyzine, lorazepam

what condition is a risk factor for chronic nephritis

diabetes mellitus

When talking with a patient about taking lubiprostone (Amitiza), the health care professional should tell the patient not to take the drug if he has which of the following? Nausea Diarrhea Urinary retention Sore throat

diarrhea rationale: Lubiprostone, a chloride channel activator, increases peristalsis and activates the chloride channels in the intestinal wall, which then increases the secretion of sodium and water. These actions treat constipation and can make diarrhea much worse.

irritable bowel syndrome

disorder of GI systems = changes in bowel function

A health care professional is assessing a client who was administered ondansetron (Zofran) IV 1 hr ago. Which of the following findings is an adverse effect of this drug?' dizziness rash abdominal cramping Tardive dyskinesia

dizziness rationale: Dizziness and lightheadedness are the most common adverse effects of ondansetron. Rash Rash is an adverse effect of sulfasalazine (Azulfidine). Tardive dyskinesia Tardive dyskinesia is an adverse effect of metoclopramide (Reglan). Abdominal cramping Abdominal cramping is an adverse effect of bisacodyl (Dulcolax).

potassium supplements potassium chloride, K gluconate, K phosphate, K bicarbonate

essential for nerve impulses. Adverse: GI upset, hyperkalemia, concurrent use of K-sparing not good

black cohosh

estrogen substitute, GI, avoid in pregnancy, increase effects of HTN meds, estrogen meds, may increase hypoglycemia,

iron preperations

ferrous sulfate- PO, Iron dextran: Parenteral PO: ferrous gluconate, parenteral: ferumoxtyol- chronic kidney disease, (iron sucrose, sodium-ferric gluconate complex- used for long term dialysis)

Valsalva maneuver:

forcible exhalation against a closed glottis followed by a rise in intrathoracic pressure and subsequent possible dramatic rise in arterial pressure; may occur during straining at stool

GERD

gastric content and enzyme leaks back into the esophagus = irritating esophagus lining = delaying clearance

antibiotics: amoxicillin, bismuth, clarithromycin, metronidazole, tetracycline

get rid of H. pylori. use a combo of 2-3 meds fr 14 days, nausea and diarrhea are common

garlic

helps w/ cholesterol, vasodilates. Adverse: GI, risk of bleeding, hypoglemia in diabetes, decreases squinavir for HIV,

Feverfew

helps w/ migrains, platelet aggragation. Adverse: mild GI, increased risk of bleeding

irritable bowel syndrome complication

hemorrhoid aggravation

esophageal varices complications

hypovolemic shock

teaching for client with chronic kidney disease is understood when client states...

i will decrease my intake of foods high in phosphorus

teaching for client with chronic kidney disease, which statement indicates understanding of teaching?

i will weigh myself each morning

echinacea

immune system, decreases inflammaton, common cold, adverse: bitter taste, BI, interacts w/ TB, HIV, cancer meds

valerian

increase GABA to prevent insomnia. Adverse: drowsiness, lightheadednes, physical dependence, careful w/ mental illness

hepatitis

inflammation of liver cells

gastritis

inflammation of the lining in the stomach

peritonitis:

inflammation of the lining of the abdominal cavity

pancreatitis

inflammation of the pancreas

glucosamine

inflammation. adverse: mild GI, shellfish allergy and antiplatelet and anitcoagulant meds

A health care professional is about to administer ondansetron (Zofran) to a patient who is receiving chemotherapy. Which of the following actions should the health care professional take? (Select all that apply.) Infuse the drug 30 min prior to chemotherapy. Administer the drug when the patient reports nausea. Infuse the drug slowly over 15 min. Administer the drug immediately following chemotherapy. Repeat the dose 4 hr after chemotherapy.

infuse drug 30 min prior infuse drug slowly over 15 min repeat dose 4 hr after rationale: Infuse the drug 30 min prior to chemotherapy is correct. The health care professional should administer IV ondansetron, a serotonin antagonist, 30 min prior to chemotherapy to prevent chemotherapy-induced nausea and vomiting. When treating radiation-induced nausea and vomiting, it is appropriate to give the drug orally 1 to 2 hr prior to radiation therapy and then every 8 hr. Administer the drug when the patient reports nausea is incorrect. Ondansetron prevents nausea and vomiting from chemotherapy, radiation therapy, and anesthesia. It is inappropriate to wait until the patient reports nausea to administer the drug. Infuse the drug slowly over 15 min is correct. The health care professional should infuse ondansetron slowly over 15 min to prevent chemotherapy-induced nausea. Administer the drug immediately following chemotherapy is incorrect. Giving ondansetron immediately following chemotherapy might not allow enough time for the drug to exert its antiemetic actions before the chemotherapy exerts emetic actions. Repeat the dose 4 hr after chemotherapy is correct. For maximum effectiveness, the health care professional should administer ondansetron 4 hr and then 8 hr after chemotherapy. When treating anesthesia-induced nausea and vomiting, it is appropriate to give the drug 1 hr before anesthesia induction.

Nursing intervention for pt with chronic kidney failure lab results: BUN 196, K 7.3, and Na 152.

infuse regular insulin in dextrose 10% in water

Action taken by nurse obtaining culture and sensitivity for pt with symptoms of UTI

instruct client to initiate flow of urine before collecting the specimen

A health care professional is reviewing the medical record of a patient who is to begin taking ranitidine (Zantac). Which of the following medications interacts with ranitidine? Phenobarbital sodium (Luminal) Ketoconazole Lisinopril (Prinivil) Hydrochlorothiazide (HydroDIURIL)

ketoconazole rationale: Phenobarbital sodium (Luminal) does not interact with ranitidine. However, it interacts with a number of drugs, including anticoagulants, oral contraceptives, and anticonvulsants. Ketoconazole Ranitidine reduces the absorption of ketoconazole. Lisinopril (Prinivil) Lisinopril (Prinivil) does not interact with ranitidine. However, lisinopril interacts with lithium carbonate (Lithobid) and can cause lithium to reach toxic levels. Hydrochlorothiazide (HydroDIURIL) Hydrochlorothiazide (HydroDIURIL) does not interact with ranitidine. However, it interacts by promoting potassium loss and increases the risk of digoxin (Lanoxin) toxicity.

A health care professional is caring for an older adult patient who is about to begin taking cimetidine (Tagamet) to treat a duodenal ulcer. The health care professional should tell the patient to report which of the following adverse reactions? Lethargy Cellulitis Dry mouth Myalgia

lethargy rationale: Cimetidine, a histamine2 receptor antagonist, can cause CNS effects, such as lethargy, depression, confusion, and seizures, especially in older adults. The health care professional should tell the patient to report these effects. If they persist, ranitidine (Zantac) might be a better alternative for the patient.

osmotic laxatives: magnesium hydroxide

low dose: prevent painful elimination high dose: prepare for surgery, rapid evacuation after poison or dead parasites

arteriovenous shunt placed in left arm. What action by new nurse requires intervention of preceptor?

measuring BP in shunted arm every 4 hrs

antacids: aluminum hydroxide, aluminum carbonate, magnesium hydroxide, sodium bicarbonate, calcium carbonte

neutralize acid, PUD and GERD, can cause constipation, magnesium causes diarrhea, sodium may cause retention, magnesium can lead to toxicity in renal impaired patients. Risk C, not with GI perforation or obstruction, aluminium cannot be taken w/ warfarin and TCA- drink water

wound-ostomy-continence (WOC) nurse:

nurse specially educated in the appropriate management of fecal and urinary diversions; guides patients, their families, surgeons, and nurses by recommending appropriate use of skin, wound, ostomy, and continence products; formerly called enterostomal therapist

A health care professional should conclude that alosetron (Lotronex) therapy is effective when a patient who is taking it reports which of the following? One formed stool per day Urination without burning No nausea or vomiting Less GI reflux

one formed stool per day rationale: Alosetron, a serotonin 5-HT3 receptor antagonist, treats the diarrhea and pain of severe irritable bowel syndrome. One formed stool per day indicates symptom relief.

A health care professional should question the use of misoprostol (Cytotec) for a patient who has which of the following? A seizure disorder Rheumatoid arthritis A positive pregnancy test Heart failure

positive pregnancy rationale: Misoprostol, a prostaglandin E analog, is a pregnancy risk category X drug. It can cause uterine contractions and induce abortion; therefore, primary care providers must confirm that patients are not pregnant before prescribing the drug. Patients taking it must use contraception.

stimulant: bisocodyl

prepare for surgery, short term treatment for high dose opioid constipation

prochlorperazine

prevents emesis related to chemo, opioids, post op, PO and IV adverse: EPS, hypotension, sedation, anticholinergic effects,

A health care professional is caring for a patient who has recently had a myocardial infarction and is about to begin taking docusate sodium (Colace). The health care professional should explain that docusate sodium will have which of the following therapeutic effects? Reduces inflammation Reduces gastric acid Prevents diarrhea Prevents straining

prevents straining rationale: Docusate sodium, a stool softener, prevents straining during defecation and prevents the elevation in blood pressure that can result from straining. It also helps relieve constipation and reduce the painful elimination of hard stools.

saw palmetto

prostate symptoms, few adverse but can decrease marker to detect prostate cancer. interactions: finasteride, anti platelet, category X

mucosal protectant: sucralfate

protective layer for up to 6hr, used w/ acute duodenal ulcers and GERD, need fiber and water to prevent constipation, no systemic effects, use cautiously w/ renal failure. antacids decrease absorption. sucralfate decreases phenytoin, digoxin, warfarin, ciprofloxacin- take 4x day, 1hr before meal and at night

laxatives:

psyllium, docusate sodium, bisacodyl, magnesium hydroxide, senna, lactulose

indication of UTI

pyuria: WBC's in urine

Proton pump inhibitor: omeprazole, pantoprazole, lansoprazole, rabeprazole sodium, esomeprazole

reduce gastric acid secretion, insignificant side effects, low incidence of headache, diarrhea and nausea/vomiting, all category B except omeprazole (C). cautiously w/ pneumonia and COPD, monitor digoxin carefully, don't use w/ itraconazole, ketoconazole, atazanavir, - don't crush, chew or break sustained release capsules, active ulcers treated 4-w6wks- notify if coffee ground emesis

magnesium sulfate: magnesium hydroxide, magnesium oxide, magnesium citrate

regulating muscle contraction and blood coagulation. Adverse: muscle weakness, painful muscles, cardiac, respiratory depression, diarrhea

surfactant: docusate

relieve constipation due to opioid use, decrease risk of fecal impaction, reduce strain.

cirrhosis

scarring of the liver

adverse of scopolamine and dimenhydrinate

sedation, anticholinergic effects, monitor I&O

St. Johns wort

serotonin, topical for infection: adverse: dry mouth, GI, skin rash in sunlight. Interactoins: antidepressant, amphetamine, cocaine, decrease effectiveness of BC, cyclosprine, warfarin, digoxin, calcium-channel blockers, steroids, HIV and some chemo meds

what should be included in teaching for pt with urge urinary incontinence?

set an interval for toileting based on previous voiding pattern

kava

should be avoided- liver injury, promotes sleep, decreases anxiety, chronic use causes jaundice, dry skin, can cause sedation w/ CNS depressants

aprepitant

should be used in combo w/ glucocorticoid to prevent CINV, nausea, vomiting, PO or IV, adverse: headache, liver damage, diarrhea, dizziness

When administering sulfasalazine (Azulfidine) to a patient, the health care professional should check which of the following to help identify an adverse effect of the drug? (Select all that apply.) Level of consciousness Skin integrity Temperature Urine output CBC

skin temp cbc Level of consciousness is incorrect. Sulfasalazine, a 5-aminosalicylate, is unlikely to affect level of consciousness. Metoclopramide (Reglan), a dopamine antagonist, is a GI-system drug that can cause sedation. Skin integrity is correct. Sulfasalazine can cause a skin rash, so the health care professional should check the patient's skin for rashes. The drug can also cause nausea. If the patient reports nausea, the health care professional should suggest taking the drug with food or water. Temperature is correct. Sulfasalazine can cause a fever, so the health care professional should check the patient's temperature and treat fever with an antipyretic. Urine output is incorrect. Sulfasalazine is unlikely to affect urine output, although it can cause an orange-yellow discoloration of urine and skin. CBC is correct. Sulfasalazine can cause hematologic disorders, such as agranulocytosis and hemolytic and macrocytic anemia. The health care professional should check the patient's CBC periodically during therapy, and tell her to report sore throat or fatigue.

aloe

sooths skin, interacts w/ digoxin, diuretics, corticosteroids, antidysrhythmics

monitoring for adverse effects of prednisone for pt with nephrotic syndrome. what should be reported to provider?

sore throat

treatment of calcium phosphate kidney stones following ESWL. what action should nurse take?

strain all urine

A health care professional is caring for a patient who takes phenytoin (Dilantin) for a seizure disorder and is about to begin sucralfate (Carafate) therapy to treat a duodenal ulcer. The health care professional should tell the patient to take the drugs at least 2 hr apart because phenytoin increases the metabolism of sucralfate. phenytoin reduces the effectiveness of sucralfate. sucralfate increases the risk for phenytoin toxicity. sucralfate interferes with the absorption of phenytoin.

sucralfate interferes with the absorption of phenytoin. rationale: Sucralfate decreases the absorption of phenytoin. The health care professional should tell the patient to allow at least 2 hr between taking the two drugs and should monitor the patient's phenytoin levels.

When talking with a patient about taking omeprazole (Prilosec) to treat a duodenal ulcer, the health care professional should include which of the following instructions? take with food swallow capsule whole dissolve tablets in water take drug at bedtime

swallow capsule whole rationale: Omeprazole, a proton pump inhibitor, is unstable in stomach acid. The health care professional should tell the patient to swallow the capsules or tablets whole and not chew the delayed-release tablets.

esophageal varices

swollen, fragile blood vessels in the esophagus

chronic kidney disease, what finding is expected?

tachypnea

following a kidney transplant, what indicates possible delay of function of transplanted kidney

urine output 30ml/2hr

cannabinoids: dronabinol

used for CINV and appetite increase in AIDS, PO adverse: avoid in mental health and cardio patients

lorazepam

used in combo for CINV

glucocorticoid: dexamethasone

used in combo w/ other antiemetic, PO/IV,

Iron info:

used to prevent anemia, adverse: GI, teeth staining, staining of skin, anaphylaxis, hypotension, fatal iron toxicity- give deferoxamine, don't take w/ antacids, tetracyclines, Vit. C increases absorption but also GI upset

histamine receptor antagonists: ranitidine hydrochloride, cimetidine, famotidine, nizatidine

used w/ antibiotics for H. pylori, suppress secretion of gastric acid; cimetidine: androgen issues and libido also CNS effects, avoid use in elderly. pregnancy risk B- monitor for bleeding w/ warfarin, antacids reduce absorption

prostagladin E analog: misoprostol

used w/ long term NSAID use to prevent ulcers, causes cervical ripening- use w/ magnesium may cause diarrhea, may have spotting or dysmenorrhea- notify provider, RISK X, take with meals and at bedtime

probiotics

various preparations to help w/ IBS, ulcerative colitis, C. Diff- Adverse: flatulence and bloating

gingko biloba

vasodilate, may decrease thrombosis, decreases bronchospasm, improves memory. Adverse: Mild GI upset, headache, lightheadedness, careful w/ seizure risk and antidepressants

priority assessment 1hour after hemodialysis

vital signs

intervention for acute glomerulonephritis

weight daily

Ma Huang

weight loss, increase HR and BP, bronchodilates- can cause death at high doses also may casue euphoria and pyschosis. interacts w/ MAOI, CNS stimulants, antihypertensives

male pt with continent internal ileal reservoir following surgery to bladder cancer. what statement should be included in teaching?

you must insert a catheter throught your stoma to drain the urine

kidney transplant 2 days ago. education regarding cyclosporine. appropriate statement by nurse:

you will need to continue taking this medication to protect your new kidneys

A 67-year-old client is returning for a follow-up appointment to the primary care group where you practice nursing. At his last appointment, he received the diagnosis of portal hypertension. What is the primary aim of portal hypertension treatment? Select all that apply. a) Reduce fluid output. b) Reduce fluid accumulation. c) Reduce venous pressure. d) Reduce blood coagulation.

• Reduce venous pressure. • Reduce fluid accumulation. Methods of treating portal hypertension aim to reduce venous pressure and fluid accumulation.


Kaugnay na mga set ng pag-aaral

Extension of the Vertebral Column: Synergist & Antagonist Muscles

View Set

Students With Disabilities CST NYSTCE (060)

View Set

Photosynthesis and celluar respiration

View Set